2002年入試問題研究

2002 年入試問題研究
2015 年 5 月 23 日
いくつかの大学の理系全問題と,その他,高校数学に意味のある「数学の問題」を中心に順次紹
介します.また新たな別解が出来たときにもここで紹介します.問題の難易ではなく,数学の問題
としての質的な面から,私の考えを述べていきます (南海).
1
目次
1
大学別
1.1
1.2
1.3
1.4
1.5
1.6
1.1.1
. . . . . . . . . . . . . . . . . . . . . . . . . . . . . . . . . . . . . . . . .
6
6
6
1.1.2
1.1.3
1.1.4
. . . . . . . . . . . . . . . . . . . . . . . . . . . . . . . . . . . . . . . . .
. . . . . . . . . . . . . . . . . . . . . . . . . . . . . . . . . . . . . . . . .
. . . . . . . . . . . . . . . . . . . . . . . . . . . . . . . . . . . . . . . . .
6
6
7
1.1.5
1.1.6
. . . . . . . . . . . . . . . . . . . . . . . . . . . . . . . . . . . . . . . . .
. . . . . . . . . . . . . . . . . . . . . . . . . . . . . . . . . . . . . . . . .
7
7
京大前期理系解答 . . . . . . . . . . . . . . . . . . . . . . . . . . . . . . . . . . . .
京大前期理系問題 . . . . . . . . . . . . . . . . . . . . . . . . . . . . . . . . . . . .
1.2.1
1.2.2
. . . . . . . . . . . . . . . . . . . . . . . . . . . . . . . . . . . . . . . . .
. . . . . . . . . . . . . . . . . . . . . . . . . . . . . . . . . . . . . . . . .
7
7
8
1.2.3
1.2.4
. . . . . . . . . . . . . . . . . . . . . . . . . . . . . . . . . . . . . . . . .
. . . . . . . . . . . . . . . . . . . . . . . . . . . . . . . . . . . . . . . . .
9
10
1.2.5
. . . . . . . . . . . . . . . . . . . . . . . . . . . . . . . . . . . . . . . . .
1.2.6
. . . . . . . . . . . . . . . . . . . . . . . . . . . . . . . . . . . . . . . . .
東大前期理系問題 . . . . . . . . . . . . . . . . . . . . . . . . . . . . . . . . . . . .
11
12
13
1.3.1
1.3.2
. . . . . . . . . . . . . . . . . . . . . . . . . . . . . . . . . . . . . . . . .
. . . . . . . . . . . . . . . . . . . . . . . . . . . . . . . . . . . . . . . . .
13
13
1.3.3
1.3.4
1.3.5
. . . . . . . . . . . . . . . . . . . . . . . . . . . . . . . . . . . . . . . . .
. . . . . . . . . . . . . . . . . . . . . . . . . . . . . . . . . . . . . . . . .
. . . . . . . . . . . . . . . . . . . . . . . . . . . . . . . . . . . . . . . . .
13
14
14
1.3.6
. . . . . . . . . . . . . . . . . . . . . . . . . . . . . . . . . . . . . . . . .
東大前期理系解答 . . . . . . . . . . . . . . . . . . . . . . . . . . . . . . . . . . . .
14
15
1.4.1
1.4.2
1.4.3
. . . . . . . . . . . . . . . . . . . . . . . . . . . . . . . . . . . . . . . . .
. . . . . . . . . . . . . . . . . . . . . . . . . . . . . . . . . . . . . . . . .
. . . . . . . . . . . . . . . . . . . . . . . . . . . . . . . . . . . . . . . . .
15
15
16
1.4.4
1.4.5
. . . . . . . . . . . . . . . . . . . . . . . . . . . . . . . . . . . . . . . . .
. . . . . . . . . . . . . . . . . . . . . . . . . . . . . . . . . . . . . . . . .
17
18
1.4.6
. . . . . . . . . . . . . . . . . . . . . . . . . . . . . . . . . . . . . . . . .
1.5.1
. . . . . . . . . . . . . . . . . . . . . . . . . . . . . . . . . . . . . . . . .
19
20
21
1.5.2
1.5.3
. . . . . . . . . . . . . . . . . . . . . . . . . . . . . . . . . . . . . . . . .
. . . . . . . . . . . . . . . . . . . . . . . . . . . . . . . . . . . . . . . . .
21
21
1.5.4
. . . . . . . . . . . . . . . . . . . . . . . . . . . . . . . . . . . . . . . . .
1.5.5
. . . . . . . . . . . . . . . . . . . . . . . . . . . . . . . . . . . . . . . . .
阪大前期理系解答 . . . . . . . . . . . . . . . . . . . . . . . . . . . . . . . . . . . .
21
22
22
1.6.1
1.6.2
. . . . . . . . . . . . . . . . . . . . . . . . . . . . . . . . . . . . . . . . .
. . . . . . . . . . . . . . . . . . . . . . . . . . . . . . . . . . . . . . . . .
22
22
1.6.3
1.6.4
1.6.5
. . . . . . . . . . . . . . . . . . . . . . . . . . . . . . . . . . . . . . . . .
. . . . . . . . . . . . . . . . . . . . . . . . . . . . . . . . . . . . . . . . .
. . . . . . . . . . . . . . . . . . . . . . . . . . . . . . . . . . . . . . . . .
24
25
26
阪大前期理系問題 . . . . . . . . . . . . . . . . . . . . . . . . . . . . . . . . . . . .
2
1.7
1.8
1.9
2
3
神大前期理系問題 . . . . . . . . . . . . . . . . . . . . . . . . . . . . . . . . . . . .
1.7.1
. . . . . . . . . . . . . . . . . . . . . . . . . . . . . . . . . . . . . . . . .
27
27
1.7.2
1.7.3
. . . . . . . . . . . . . . . . . . . . . . . . . . . . . . . . . . . . . . . . .
. . . . . . . . . . . . . . . . . . . . . . . . . . . . . . . . . . . . . . . . .
27
28
1.7.4
. . . . . . . . . . . . . . . . . . . . . . . . . . . . . . . . . . . . . . . . .
1.7.5
. . . . . . . . . . . . . . . . . . . . . . . . . . . . . . . . . . . . . . . . .
神大前期理系解答 . . . . . . . . . . . . . . . . . . . . . . . . . . . . . . . . . . . .
28
28
29
1.8.1
1.8.2
. . . . . . . . . . . . . . . . . . . . . . . . . . . . . . . . . . . . . . . . .
. . . . . . . . . . . . . . . . . . . . . . . . . . . . . . . . . . . . . . . . .
29
30
1.8.3
1.8.4
1.8.5
. . . . . . . . . . . . . . . . . . . . . . . . . . . . . . . . . . . . . . . . .
. . . . . . . . . . . . . . . . . . . . . . . . . . . . . . . . . . . . . . . . .
. . . . . . . . . . . . . . . . . . . . . . . . . . . . . . . . . . . . . . . . .
30
31
32
京大後期理系問題 . . . . . . . . . . . . . . . . . . . . . . . . . . . . . . . . . . . .
1.9.1
. . . . . . . . . . . . . . . . . . . . . . . . . . . . . . . . . . . . . . . . .
32
33
1.9.2
1.9.3
1.9.4
. . . . . . . . . . . . . . . . . . . . . . . . . . . . . . . . . . . . . . . . .
. . . . . . . . . . . . . . . . . . . . . . . . . . . . . . . . . . . . . . . . .
. . . . . . . . . . . . . . . . . . . . . . . . . . . . . . . . . . . . . . . . .
33
33
33
1.9.5
1.9.6
. . . . . . . . . . . . . . . . . . . . . . . . . . . . . . . . . . . . . . . . .
. . . . . . . . . . . . . . . . . . . . . . . . . . . . . . . . . . . . . . . . .
33
34
1.10 京大後期理系解答 . . . . . . . . . . . . . . . . . . . . . . . . . . . . . . . . . . . .
1.10.1
. . . . . . . . . . . . . . . . . . . . . . . . . . . . . . . . . . . . . . . . .
1.10.2
. . . . . . . . . . . . . . . . . . . . . . . . . . . . . . . . . . . . . . . . .
34
34
34
1.10.3
1.10.4
1.10.5
. . . . . . . . . . . . . . . . . . . . . . . . . . . . . . . . . . . . . . . . .
. . . . . . . . . . . . . . . . . . . . . . . . . . . . . . . . . . . . . . . . .
. . . . . . . . . . . . . . . . . . . . . . . . . . . . . . . . . . . . . . . . .
35
36
37
1.10.6
. . . . . . . . . . . . . . . . . . . . . . . . . . . . . . . . . . . . . . . . .
38
分野別
39
数
39
3.1
3.2
問題 . . . . . . . . . . . . . . . . . . . . . . . . . . . . . . . . . . . . . . . . . . .
3.1.1
3.1.2
名大前期理系 . . . . . . . . . . . . . . . . . . . . . . . . . . . . . . . . . .
3.1.3
3.1.4
九大前期理系 . . . . . . . . . . . . . . . . . . . . . . . . . . . . . . . . . .
名大前期文系 . . . . . . . . . . . . . . . . . . . . . . . . . . . . . . . . . .
39
39
39
慶応医 4 番 . . . . . . . . . . . . . . . . . . . . . . . . . . . . . . . . . . .
39
40
3.1.5 金沢前期理系 . . . . . . . . . . . . . . . . . . . . . . . . . . . . . . . . . .
3.1.6 金沢後期理系 . . . . . . . . . . . . . . . . . . . . . . . . . . . . . . . . . .
解答 . . . . . . . . . . . . . . . . . . . . . . . . . . . . . . . . . . . . . . . . . . .
40
41
42
3.2.1
3.2.2
名大前期理系 . . . . . . . . . . . . . . . . . . . . . . . . . . . . . . . . . .
42
42
3.2.3
3.2.4
3.2.5
九大前期理系 . . . . . . . . . . . . . . . . . . . . . . . . . . . . . . . . . .
名大前期文系 . . . . . . . . . . . . . . . . . . . . . . . . . . . . . . . . . .
慶応医 4 番 . . . . . . . . . . . . . . . . . . . . . . . . . . . . . . . . . . .
金沢前期理系 . . . . . . . . . . . . . . . . . . . . . . . . . . . . . . . . . .
3
43
44
46
金沢後期理系 . . . . . . . . . . . . . . . . . . . . . . . . . . . . . . . . . .
47
問題 . . . . . . . . . . . . . . . . . . . . . . . . . . . . . . . . . . . . . . . . . . .
48
48
48
3.2.6
4
式
4.1
4.2
5
横国大後期理系 . . . . . . . . . . . . . . . . . . . . . . . . . . . . . . . . .
4.1.2
4.1.3
京都府立医大 . . . . . . . . . . . . . . . . . . . . . . . . . . . . . . . . . .
慶応理工 . . . . . . . . . . . . . . . . . . . . . . . . . . . . . . . . . . . . .
48
48
4.1.4 岐阜後期理系 . . . . . . . . . . . . . . . . . . . . . . . . . . . . . . . . . .
解答 . . . . . . . . . . . . . . . . . . . . . . . . . . . . . . . . . . . . . . . . . . .
4.2.1 横国大後期理系 . . . . . . . . . . . . . . . . . . . . . . . . . . . . . . . . .
49
49
49
4.2.2
4.2.3
京都府立医大 . . . . . . . . . . . . . . . . . . . . . . . . . . . . . . . . . .
慶応理工 . . . . . . . . . . . . . . . . . . . . . . . . . . . . . . . . . . . . .
50
52
4.2.4
岐阜後期理系 . . . . . . . . . . . . . . . . . . . . . . . . . . . . . . . . . .
52
図形
5.1
5.2
6
4.1.1
54
問題 . . . . . . . . . . . . . . . . . . . . . . . . . . . . . . . . . . . . . . . . . . .
5.1.1
5.1.2
北大前期理系 . . . . . . . . . . . . . . . . . . . . . . . . . . . . . . . . . .
5.1.3
5.1.4
九大前期理系 . . . . . . . . . . . . . . . . . . . . . . . . . . . . . . . . . .
5.1.5
5.1.6
5.1.7
一橋 . . . . . . . . . . . . . . . . . . . . . . . . . . . . . . . . . . . . . . .
名大前期文系 . . . . . . . . . . . . . . . . . . . . . . . . . . . . . . . . . .
広大後期理 . . . . . . . . . . . . . . . . . . . . . . . . . . . . . . . . . . .
福井医大 . . . . . . . . . . . . . . . . . . . . . . . . . . . . . . . . . . . . .
解答 . . . . . . . . . . . . . . . . . . . . . . . . . . . . . . . . . . . . . . . . . . .
56
56
5.2.2
5.2.3
5.2.4
東北大前期理系 . . . . . . . . . . . . . . . . . . . . . . . . . . . . . . . . .
5.2.5
5.2.6
一橋 . . . . . . . . . . . . . . . . . . . . . . . . . . . . . . . . . . . . . . .
広大後期理 . . . . . . . . . . . . . . . . . . . . . . . . . . . . . . . . . . .
60
62
5.2.7
福井医大 . . . . . . . . . . . . . . . . . . . . . . . . . . . . . . . . . . . . .
64
九大前期理系 . . . . . . . . . . . . . . . . . . . . . . . . . . . . . . . . . .
名大前期文系 . . . . . . . . . . . . . . . . . . . . . . . . . . . . . . . . . .
57
58
59
65
問題 . . . . . . . . . . . . . . . . . . . . . . . . . . . . . . . . . . . . . . . . . . .
東大理系後期 . . . . . . . . . . . . . . . . . . . . . . . . . . . . . . . . . .
お茶大前期 . . . . . . . . . . . . . . . . . . . . . . . . . . . . . . . . . . .
解答 . . . . . . . . . . . . . . . . . . . . . . . . . . . . . . . . . . . . . . . . . . .
65
65
66
6.2.1
東大理系後期 . . . . . . . . . . . . . . . . . . . . . . . . . . . . . . . . . .
66
66
6.2.2
お茶大前期 . . . . . . . . . . . . . . . . . . . . . . . . . . . . . . . . . . .
69
確率
7.1
55
55
56
北大前期理系 . . . . . . . . . . . . . . . . . . . . . . . . . . . . . . . . . .
6.1.1
6.1.2
6.2
55
55
5.2.1
数列
6.1
7
東北大前期理系 . . . . . . . . . . . . . . . . . . . . . . . . . . . . . . . . .
54
54
54
70
問題 . . . . . . . . . . . . . . . . . . . . . . . . . . . . . . . . . . . . . . . . . . .
7.1.1
7.1.2
東大理系後期総合 II . . . . . . . . . . . . . . . . . . . . . . . . . . . . . .
九大前期理系 . . . . . . . . . . . . . . . . . . . . . . . . . . . . . . . . . .
4
70
70
71
7.2
8
7.1.3
7.1.4
大阪女子大前期理系 . . . . . . . . . . . . . . . . . . . . . . . . . . . . . .
7.1.5
7.1.6
名大後期理系 . . . . . . . . . . . . . . . . . . . . . . . . . . . . . . . . . .
北大前期理系 . . . . . . . . . . . . . . . . . . . . . . . . . . . . . . . . . .
慶応医 2 番 . . . . . . . . . . . . . . . . . . . . . . . . . . . . . . . . . . .
解答 . . . . . . . . . . . . . . . . . . . . . . . . . . . . . . . . . . . . . . . . . . .
7.2.1
7.2.2
東大理系後期総合 II . . . . . . . . . . . . . . . . . . . . . . . . . . . . . .
7.2.3
7.2.4
大阪女子大前期理系 . . . . . . . . . . . . . . . . . . . . . . . . . . . . . .
7.2.5
7.2.6
名大後期理系 . . . . . . . . . . . . . . . . . . . . . . . . . . . . . . . . . .
九大前期理系 . . . . . . . . . . . . . . . . . . . . . . . . . . . . . . . . . .
北大前期理系 . . . . . . . . . . . . . . . . . . . . . . . . . . . . . . . . . .
慶応医 2 番 . . . . . . . . . . . . . . . . . . . . . . . . . . . . . . . . . . .
曲線
8.1
問題 . . . . . . . . . . . . . . . . . . . . . . . . . . . . . . . . . . . . . . . . . . .
8.1.1
8.2
九大前期理系 . . . . . . . . . . . . . . . . . . . . . . . . . . . . . . . . . .
解答 . . . . . . . . . . . . . . . . . . . . . . . . . . . . . . . . . . . . . . . . . . .
72
72
72
73
73
73
74
75
77
77
79
80
80
80
九大前期理系 . . . . . . . . . . . . . . . . . . . . . . . . . . . . . . . . . .
81
81
行列
9.1 問題 . . . . . . . . . . . . . . . . . . . . . . . . . . . . . . . . . . . . . . . . . . .
82
82
9.1.1 金沢大理系後期 . . . . . . . . . . . . . . . . . . . . . . . . . . . . . . . . .
9.1.2 富山医科薬科 . . . . . . . . . . . . . . . . . . . . . . . . . . . . . . . . . .
解答 . . . . . . . . . . . . . . . . . . . . . . . . . . . . . . . . . . . . . . . . . . .
82
83
83
金沢大理系後期 . . . . . . . . . . . . . . . . . . . . . . . . . . . . . . . . .
富山医科薬科 . . . . . . . . . . . . . . . . . . . . . . . . . . . . . . . . . .
83
84
10 微積
10.1 問題 . . . . . . . . . . . . . . . . . . . . . . . . . . . . . . . . . . . . . . . . . . .
86
86
10.1.1 千葉大理系後期 . . . . . . . . . . . . . . . . . . . . . . . . . . . . . . . . .
10.1.2 東工大前期 . . . . . . . . . . . . . . . . . . . . . . . . . . . . . . . . . . .
10.1.3 東北大前期理系 . . . . . . . . . . . . . . . . . . . . . . . . . . . . . . . . .
86
86
86
10.1.4 名大前期理系 . . . . . . . . . . . . . . . . . . . . . . . . . . . . . . . . . .
10.2 解答 . . . . . . . . . . . . . . . . . . . . . . . . . . . . . . . . . . . . . . . . . . .
87
87
10.2.1 千葉大理系後期 . . . . . . . . . . . . . . . . . . . . . . . . . . . . . . . . .
10.2.2 東工大前期 . . . . . . . . . . . . . . . . . . . . . . . . . . . . . . . . . . .
10.2.3 東北大前期理系 . . . . . . . . . . . . . . . . . . . . . . . . . . . . . . . . .
87
88
90
10.2.4 名大前期理系 . . . . . . . . . . . . . . . . . . . . . . . . . . . . . . . . . .
10.3 高校数学の方法関連問題 . . . . . . . . . . . . . . . . . . . . . . . . . . . . . . . .
90
91
10.3.1 何をどう置くか . . . . . . . . . . . . . . . . . . . . . . . . . . . . . . . . .
10.3.2 条件の同値変形 . . . . . . . . . . . . . . . . . . . . . . . . . . . . . . . . .
10.3.3 例で考える . . . . . . . . . . . . . . . . . . . . . . . . . . . . . . . . . . .
91
91
92
10.3.4 必要条件・十分条件 . . . . . . . . . . . . . . . . . . . . . . . . . . . . . .
10.3.5 存在の証明 . . . . . . . . . . . . . . . . . . . . . . . . . . . . . . . . . . .
92
92
10.3.6 定義を問う問題 . . . . . . . . . . . . . . . . . . . . . . . . . . . . . . . . .
93
8.2.1
9
9.2
9.2.1
9.2.2
5
10.3.7 背理法 . . . . . . . . . . . . . . . . . . . . . . . . . . . . . . . . . . . . . .
10.4 高校数学の方法関連問題解答 . . . . . . . . . . . . . . . . . . . . . . . . . . . . . .
94
95
10.4.1 何をどう置くか . . . . . . . . . . . . . . . . . . . . . . . . . . . . . . . . .
10.4.2 条件の同値変形 . . . . . . . . . . . . . . . . . . . . . . . . . . . . . . . . .
95
95
10.4.3 例で考える . . . . . . . . . . . . . . . . . . . . . . . . . . . . . . . . . . .
10.4.4 必要条件・十分条件 . . . . . . . . . . . . . . . . . . . . . . . . . . . . . .
10.4.5 存在の証明 . . . . . . . . . . . . . . . . . . . . . . . . . . . . . . . . . . .
96
97
98
10.4.6 定義を問う問題 . . . . . . . . . . . . . . . . . . . . . . . . . . . . . . . . .
10.4.7 背理法 . . . . . . . . . . . . . . . . . . . . . . . . . . . . . . . . . . . . . .
99
99
大学別
1
1.1
京大前期理系問題
今年の京大前期理系の問題は平凡なものばかりだった.
「難易度」が易という意味ではない.問題
自体のおもしろさがなく,よくある問題の外見を少し変えただけというものばかりであった.こつ
こつ勉強してきた人にとっては肩すかしのように感じられたかも知れない.
例年は少なくとも 1,2 題,高校数学への問題提起となる出題がなされていた.高校生がこんな
ことが成り立つのかと感心するような新鮮な事例を提供する問題もあった.今年はそれがなかった.
入学試験はもちろん選拔のためであるが,同時に高校数学に対する大学としての見解の具体的な
表明であり,また,高校生に考える材料を提供するという意味もある.だから,それなりの工夫と
研究が必要だし,責任もある.
今後に期待したい.
1.1.1
数列 {an } の初項 a1 から第 n 項 an までの和を Sn と表す.この数列が
a1 = 1, lim Sn = 1, n(n − 2)an+1 = Sn (n >
= 1)
n→∞
を満たすとき,一般項 an を求めよ.
1.1.2
半径 1 の円周上に相異なる 3 点 A, B, C がある.
(1) AB2 + BC2 + CA2 > 8 ならば △ABC は鋭角三角形であることを示せ.
(2) AB2 + BC2 + CA2 <
= 9 が成立することを示せ.また,この場合等号が成立するのはどのよ
うな場合か.
1.1.3
f (x) = x4 + ax3 + bx3 + cx + 1 は整数を係数とする x の 4 次式である.4 次方程式 f (x) = 0
の重複も込めた 4 つの解のうち,2 つは整数で殘りの 2 つは虚数であるという.このとき a, b, c
の値を求めよ.
6
1.1.4
(1) x >
= 0 で定義された関数 f (x) = log(x +
√
1 + x2 ) について,導関数 f ′ (x) を求めよ.
(2) 極方程式 r = θ (θ >
= 0) で定義される曲線の, 0 <
=θ<
= π の部分の長さを求めよ.
1.1.5
a, b, c を実数とする. y = x3 + 3ax2 + 3bx と y = c のグラフが相異なる 3 つの交点を持つ
という.このとき a2 > b が成立することを示し,さらにこれらの交点の x 座標のすべては開区間
√
√
(−a − 2 a2 − b, −a + 2 a2 − b) に含まれていることを示せ.
1.1.6
0 < θ < 90 とし, a は正の定数とする.複素数平面上の点 z0 , z1 , z2 · · · を次の条件 (i),(ii) を
満たすように定める.
(i) z0 = 0, z1 = a
◦
(ii) n >
= 1 のとき,点 zn − zn−1 の回りに θ 回転すると点 zn+1 − zn に一致する.
このとき点 zn (n >
= 1) が点 z0 と一致するような n が存在するための必要十分条件は,θ が有理
数であることを示せ.
1.2
京大前期理系解答
1.2.1
n>
= 1 に対し an+1 = Sn+1 − Sn である.ゆえに n(n − 2)(Sn+1 − Sn ) = Sn が成り立つ.これ
から
n(n − 2)Sn+1 = (n − 1)2 Sn
n>
= 3 のとき,
n
n−1
Sn+1 =
Sn
n−1
n−2
したがって
n−1
2
Sn = S3
n−2
1
ここで lim Sn = 1 なので,
n→∞
n−1
Sn = 1
n→∞ n − 2
lim
したがって
2
1
S3 = 1 となり, S3 =
1
2
∴
Sn =
n−2
(n >
= 3)
n−1
7
···⃝
1
つまり n >
= 4 のとき
an = Sn − Sn−1 =
n−2 n−3
1
−
=
n−1 n−2
(n − 1)(n − 2)
一方⃝
1 で n = 2 とすれば S2 = 0 .S3 =
···⃝
2
1
とあわせて
2
a2 = −1, a3 =
1
2
n = 3 のときは⃝
2 式に含まれる.
∴
a1 = 1, a2 = −1, an =
1
(n >
= 3)
(n − 1)(n − 2)
1.2.2
(1) △ABC の各頂角を α, β, γ と置く.また △ABC を A(1, 0) として座標平面上において一
般性 を失わない.
このとき鋭角三角形でも鈍角三角形でも B(cos 2γ, sin 2γ) , C(cos 2(γ + α), sin 2(γ + α)) と
なる.
AB2 + BC2 + CA2
= (cos 2γ − 1)2 + sin2 2γ + (cos 2(γ + α) − cos 2γ)2
+(sin 2(γ + α) − sin 2γ)2 + (cos 2(γ + α) − 1)2 + sin2 2(γ + α)
= 2 − 2 cos 2γ + 2
−2{cos 2(γ + α) cos 2γ + sin 2(γ + α) sin 2γ} + 2 − 2 cos 2(γ + α)
= 6 − 2{cos 2γ + cos 2α + cos 2(γ + α)}
= 6 − 2{2 cos(γ + α) cos(γ − α) + 2 cos2 (γ + α) − 1}
= 8 − 4 cos(γ + α){cos(γ − α) + cos(γ + α)}
= 8 − 8 cos(γ + α) cos γ cos α
ゆえに AB2 + BC2 + CA2 > 8 は cos(γ + α) cos γ cos α < 0 を意味する.
π
π
π
三角形の内角なので α, γ がともに α, γ >
= 2 となることは起こらない.α >
= 2 または γ >
= 2
π
なら π > γ + α >
= 2 となり cos(γ + α) cos γ cos α >
= 0.
ゆえに cos(γ + α) cos γ cos α < 0 となるのは
0 < α, γ <
π π
,
<γ+α<π
2 2
β = π − (γ + α) なので,3 つの頂角はいずれも鋭角である.
(2) (1) から △ABC が鋭角三角形のときに示せば十分である.
与不等式を示すためには,−1 <
= 8 cos(γ + α) cos γ cos α が成り立つことを示せばよい.
8
γ を固定して
f (α) = 8 cos(γ + α) cos γ cos α
と置く.このとき
f ′ (α) = −8 sin(γ + α) cos γ cos α − 8 cos(γ + α) cos γ sin α = −8 cos γ sin(2α + γ)
△ABC が鋭角三角形なので cos γ > 0 である.0 < 2α + γ < 2π なので 2α + γ = π のとき
f (α) は極小かつ最小になる.
π−γ
α=
より
2
(
)
π−γ
f
=
2
(
)
(
)
π+γ
π−γ
cos γ cos
2
2
γ
= −8 sin2 cos γ = (4 cos γ − 1) cos γ
2
(
)2
1
= 4 cos γ −
−1>
= −1
2
8 cos
1
π
π
つまり γ =
.このとき α = β =
.よって等号が成立するのは
2
3
3
△ABC が正三角形のとき.
等号成立は cos γ =
1.2.3
係数が実数なので,2 つの虚数解は共役である.整数解を m, n ,虚数解を α, α
¯ とする.
解と係数の関係から
m+n+α+α
¯ = −a · · · ⃝
1
mn + m(α + α
¯ ) + n(α + α
¯ ) + αα
¯=b
mnαα
¯=1
···⃝
2
···⃝
3
式⃝
¯ は整数.よって式⃝
¯ も整数.
1 から α + α
2 から αα
式⃝
¯=1
3 から mn = 1, αα
α = cos θ + i sin θ とすると α + α
¯ = 2 cos θ .これが整数でしかも θ ̸= 0, π なので cos θ = 0, ±
.
それぞれに応じて α, α
¯ = ±i,
√
√
1± 3 −1± 3
,
である.このときそれぞれ
2
2
(x − α)(x − α
¯ ) = x2 + 1, x2 − x + 1, x2 + x + 1
一方 mn = 1 より
∴
f (x) =
∴
(x − m)(x − n) = x2 − 2x + 1, x2 + 2x + 1


(x2 + 1)(x2 − 2x + 1) = x4 − 2x3 + 2x2 − 2x + 1





(x2 + 1)(x2 + 2x + 1) = x4 + 2x3 + 2x2 + 2x + 1



 (x2 − x + 1)(x2 − 2x + 1) = x4 − 3x3 + 4x2 − 3x + 1

(x2 − x + 1)(x2 + 2x + 1) = x4 + x3 + x + 1





(x2 + x + 1)(x2 − 2x + 1) = x4 − x3 − x + 1



 (x2 + x + 1)(x2 + 2x + 1) = x4 + 3x3 + 4x2 + 3x + 1
(a, b, c) = (±1, 0, ±1), (±2, 2, ±2), (±3, 4, ±3),
9
1
2
1.2.4
(1)
f ′ (x) =
1
√
x + 1 + x2
(
)
x
1
1+ √
=√
1 + x2
1 + x2
(2) 曲線を,極座標の極を原点,始線を x 軸の正の部分にあわせて xy 座標平面におく.このと
き極座標で (r, θ) と表される点は xy 座標では (r cos θ, r sin θ) となる.
曲線の方程式は r = θ なので,この曲線は xy 座標では媒介変数 θ によって
x = θ cos θ, y = θ sin θ
と表される.
ここで
(
dx
dθ
)2
(
+
dy
dθ
)2
= (cos θ − θ sin θ)2 + (sin θ + θ cos θ)2
= 1 + θ2
したがって求める長さを l とすると
√
∫ π ( )2 ( )2
dx
dy
l =
+
dθ
dθ
dθ
0
∫ π√
=
1 + θ2 dθ . . . ⃝
1
0
∫ π
√
=
(θ)′ 1 + θ2 dθ
0
[ √
]π ∫ π
θ2
2
√
= θ 1+θ
−
dθ
0
1 + θ2
0
}
∫ π {√
√
1
1 + θ2 − √
dθ
= π 1 + π2 −
1 + θ2
0
∫
√
1
√
(1) より
dθ = log(θ + 1 + θ2 ) + C なので
1 + θ2
[
]π
√
√
l = π 1 + π 2 − l + log(θ + 1 + θ2 )
0
√
√
2
2
= π 1 + π − l + log(π + 1 + π )
∴
l=
}
√
√
1{
log(π + 1 + π 2 ) + π 1 + π 2
2
√
[別解] ⃝
1 の計算は t = log(θ + 1 + θ2 ) と変数変換してもよい.
√
√
1
このとき θ + 1 + θ2 = et で,e−t = √
= 1 + θ2 − θ となる.
1 + θ2 + θ
∴
また (1) から
√
et + e−t
1 + θ2 =
2
√
dt
1
,つまり dθ = 1 + θ2 dt
=√
2
dθ
1+θ
10
積分域は θ : 0 → log(π +
∫
l
α
=
∫
0
α
√
√
1 + π 2 ) となるので α = log(π + 1 + π 2 ) とおく.
√
√
1 + θ2 · 1 + θ2 dt
(
=
0
et + e−t
2
)2
dt
∫
1 α 2t
(e + 2 + e−2t ) dt
4 0
[
]α
1 1 2t
1
=
e + 2t − e−2t
4 2
2
0
)
(
1 2α
−2α
=
e + 4α − e
8
}
√
√
√
1{
=
(π + 1 + π 2 )2 + 4 log(π + 1 + π 2 ) − (π + 1 + π 2 )−2
8
}
√
√
1{
=
log(π + 1 + π 2 ) + π 1 + π 2
2
√
[注意]上の計算と同様に y = log(x + 1 + x2 ) に対して
=
x=
となる.つまり関数 f (x) = log(x +
√
ey + e−y
2
1 + x2 ) と関数 g(x) =
ex + e−x
は,互いに逆関
2
数である.
1.2.5
f (x) = x3 + 3ax2 + 3bx とおく.y = f (x) と y = c のグラフが相異なる 3 つの交点を持つため
には,関数 f (x) が極大値と極小値を持つことが必要である.
f ′ (x) = 3x2 + 6ax + 3b であるから
···⃝
1
3x2 + 6ax + 3b = 3(x2 + 2ax + b) = 0
が相異なる実数解を持たねばならない.
D/4 = a2 − b > 0
∴
が成立する.
√
√
⃝
1 の 2 つの実数解を α = −a − a2 − b, β = −a + a2 − b とする.このとき f (x) は x = α
で極大, x = β で極小である.
ここで f (x) = f (α) となる α 以外の x の値を γ , f (x) = f (β) となる β 以外の x の値を δ と
する.
増減と値は次のようになる.
x
′
···
δ
···
···
α
β
···
γ
···
f (x)
+
+
+
0
−
0
+
+
+
f (x)
↗
f (β)
↗
f (α)
↘
f (β)
↗
f (α)
↗
11
ゆえに 2 つのグラフが 3 つの交点を持つのは f (β) < c < f (α) のときである.
またこのとき y = f (x) と y = c の交点の x 座標は 3 つの開区間
(δ, α), (α, β), (β, γ)
に各々1 つずつある.
f (x) − f (α) = x3 + 3ax2 + 3bx − (α3 + 3aα2 + 3bα) = 0
の解が α (重解) と γ となるので解と係数の関係から 2α + γ = −3a
√
√
ゆえに γ = −a + 2 a2 − b となる.同様に δ = −a − 2 a2 − b となる.
よって題意が示された.
1.2.6
α = cos θ + i sin θ とおく.条件から
zn+1 − zn = α(zn − zn−1 )
である.
したがって
zn+1 − zn = αn (z1 − z0 ) = aαn
α ̸= 1 であるから
zn = z0 +
n−1
∑
aαk =
k=0
a(1 − αn )
1−α
題意を満たす n が存在することは 1 − α = 0 となる n が存在することと同値である.
n
1 − αn = 0
⇐⇒
cos nθ + i sin nθ = 1
⇐⇒
nθ = 360◦ · k となる k が存在する
k
となり,θ は有理数である.
n
p
p
逆に θ が有理数であるとし,θ = 360 · とおく.nθ = 360 · k は n · = k である.これを満た
q
q
す k がとれる n として n = q が存在する.実際 k = p が条件を満たす.
このような k が存在するような n が存在すれば θ = 360 ·
よって題意が示された.
12
東大前期理系問題
1.3
昨年,これまでの東大入試数学にあった複雑な計算問題や煩瑣な場合分けは出題されず,代わっ
て「論理・論証」が文理とも出題された.明確な論証能力を問うものであった.日常的で具体的な
問題に対して数学的なモデルを立て,それに基づいて論証するという,数学本来の考える力が問わ
れた.
この傾向は今年より明確になり,計算はさらに簡単に,また基本問題が出題されるようになった.
このような傾向を積極的に理解すれば,受験生に対する東大からのメッセージであり,その意味
は「数学本来の勉強に立ち返れ」ということである.
受験数学を学問として正面から学び,考える力をつける,という青空学園数学科の方針どおり,
論証問題を中心に日頃から本格的に考え勉強してきた人には,ほぼすべて解答できる問題であった.
その意味で,出題者の意図は明確であったが,やはり何か新しい事実を提供する出題がほしかっ
た.3 番は数年前の旧課程時代の空間座標に慣れていた高校生には簡単だし,6 番は合同式の考え
方の応用として昔から知られたことである.
1.3.1
2 つの放物線
√
y = 2 3(x − cos θ)2 + sin θ
√
y = −2 3(x + cos θ)2 − sin θ
が相異なる 2 点で交わるような一般角 θ の範囲を求めよ.
1.3.2
n は正の整数とする. xn+1 を x2 − x − 1 で割った余りを
an x + bn
とおく.
(1) 数列 {an }, {bn }, n = 1, 2, · · · , は
{
an+1 = an + bn
bn+1 = an
を満たすことを示せ.
(2) n = 1, 2, · · · , に対して, an , bn は共に正の整数で,互いに素であることを証明せよ.
1.3.3
xyz 空間内の原点 O(0, 0, 0) を中心とし,点 A(0, 0, −1) を通る球面を S とする. S の外側
にある点 P(x, y, z) に対し, OP を直径とする球面と S との交わりとして得られる平面を L と
する.点 P と点 A から平面 L へ下ろした垂線の足をそれぞれ Q, R とする.このとき,
PQ <
= AR
であるような点 P の動く範囲 V を求め, V の体積は 10 より小さいことを示せ.
13
1.3.4
a は正の実数とする. xy 平面の y 軸上に点 P(0, a) をとる.関数
y=
x2
x2 + 1
のグラフを C とする. C 上の点 Q で次の条件を満たすものが原点 O(0, 0) 以外に存在するよう
な a の範囲を求めよ.
条件 : Q における C の接線が直線 PQ と直交する.
1.3.5
)
k
k
O を原点とする xyz 空間に点 Pk
, 1 − , 0 , k = 1, 2, · · · , をとる.また, z 軸上 z >
=0
n
n
の部分に,点 Qk を線分 Pk Qk の長さが 1 になるようにとる.三角錐 OPk Pk+1 Qk の体積を Vk
(
とおいて,極限
lim
n−1
∑
n→∞
Vk
k=0
を求めよ.
1.3.6
N を正の整数とする. 2N 個の項からなる数列
{a1 , a2 , · · · , aN , b1 , b2 , · · · , bN }
を
{b1 , a1 , b2 , a2 , · · · , bN , aN }
という数列に並べ替える操作を「シャッフル」と呼ぶことにする.並べ替えた数列は b1 を初項とし,
bi の次に ai , ai の次に bi+1 が来るようなものになる.また,数列 {1, 2, · · · , 2N } をシャッフ
ルしたときに得られる数列において,数 k が現れる位置を f (k) で表す.
たとえば, N = 3 のとき, {1, 2, 3, 4, 5, 6} をシャッフルすると {4, 1, 5, 2, 6, 3} となるの
で, f (1) = 2 ,f (2) = 4 ,f (3) = 6 ,f (4) = 1 ,f (5) = 3 ,f (6) = 5 である.
(1) 数列 {1, 2, 3, 4, 5, 6, 7, 8} を 3 解シャッフルしたときに得られる数列を求めよ.
(2) 1 <
= 2N を満たす任意の整数 k に対し, f (k) − 2k は 2N + 1 で割りきれることを示せ.
=k<
(3) n を正の整数とし, N = 2n−1 のときを考える.数列 {1, 2, 3, · · · , 2N } を 2n 回シャッフ
ルすると,{1, 2, 3, · · · , 2N } に戻ることを証明せよ.
14
1.4
東大前期理系解答
1.4.1
√
√
2 3(x − cos θ)2 + sin θ = −2 3(x + cos θ)2 − sin θ
が相異なる 2 つの実数解を持てばよい.これを整理する.
√
√
2 3x2 = −(2 3 cos2 θ + sin θ)
ゆえに,求める角 θ の条件は
√
√
√
−(2 3 cos2 θ + sin θ) = 2 3 sin2 θ − sin θ + 2 3 > 0
つまり
(2 sin θ +
ここでつねに
√ √
3)( 3 sin θ − 2) > 0
√
3 sin θ − 2 < 0 なので,条件は
2 sin θ +
∴
√
3<0
4
5
π + 2nπ < θ < π + 2nπ (n : 整数)
3
3
1.4.2
(1) xn+1 を x2 − x − 1 で割った商を Qn (x) とすると,各 n に対して
xn+1 = Qn (x)(x2 − x − 1) + an x + bn
となる.したがって
Qn+1 (x)(x2 − x − 1) + an+1 x + bn+1 = xn+2
= x(xn+1 ) = x{Qn (x)(x2 − x − 1) + an x + bn }
= xQn (x)(x2 − x − 1) + an (x2 − x − 1) + (an + bn )x + an
割り算の一意性より,
an+1 x + bn+1 = (an + bn )x + an
これは恒等式であるから係数を比較して,
{
an+1 = an + bn
bn+1 = an
が成立する.
15
(2) an , bn が共に互いに素な正の整数であることを数学的帰納法で示す.
x2 = x2 − x − 1 + (x + 1)
より a1 = 1, b1 = 1 .ゆえに n = 1 のとき,成立する.
n のとき成立するとする.
(1) より an+1 , bn+1 は正の整数である.
an+1 , bn+1 の最大公約数を d とし, an+1 = da, bn+1 = db とおく.
(1) より
an = bn+1 = db, bn = an+1 − bn+1 = d(a − b)
なので, d は an , bn の公約数である.帰納法の仮定から an , bn は互いに素なので, d = 1
.つまり an+1 , bn+1 も互いに素である.したがって n + 1 のときにも成立する.
ゆえに,一般に自然数 n に対して題意が成立することが示された.
1.4.3
xyz 空間の平面 α とその上にない点 M(x1 , y1 , z1 ) との距離を求める.
平面 α と直交するベクトル ⃗n = (a, b, c) をとる.また α 上の点 (x0 , y0 , z0 ) をとる.このと
きαは
a(x − x0 ) + b(y − y0 ) + c(z − z0 ) = 0
を満たす点 (x, y, z) の集合である. d = −(ax0 + by0 + cz0 ) とおき α を
ax + by + cz + d = 0
と表す.
点 M から平面 α への垂線の足を H(x2 , y2 , z2 ) とする.
−−→
MH = t⃗n
とかける.
(x2 , y2 , z2 ) = (x1 , y1 , z1 ) + (ta, tb, tc)
この (x2 , y2 , z2 ) は α 上の点なので
ax2 + by2 + cz2 + d = 0
つまり
t(a2 + b2 + c2 ) + ax1 + by1 + cz1 + d = 0
ゆえに
√
−−→
|MH| = |t⃗n| = |t| a2 + b2 + c2
|ax1 + by1 + cz1 + d|
|ax1 + by1 + cz1 + d| √ 2
√
· a + b2 + c2 =
=
a2 + b2 + c 2
a2 + b2 + c 2
16
さて球面 S は
x2 + y 2 + z 2 = 1
と表される.点 P の座標を (X, Y, Z) とする. OP を直径とする球面は
(
x−
X
2
)2
(
)2 (
)2 ( )2 ( )2 ( )2
Y
Z
X
Y
Z
+ y−
+ z−
=
+
+
2
2
2
2
2
つまり
x2 − Xx + y 2 − Y y + z 2 − Zz = 0
と表される.ゆえに S との交わりの平面 L は両式を満たす点の集合なので
L : Xx + Y y + Zz − 1 = 0
ゆえに
| − Z − 1|
AR = √
,
X2 + Y 2 + Z2
PQ =
|X 2 + Y 2 + Z 2 − 1|
√
X2 + Y 2 + Z2
したがって条件 PQ <
= AR は
|X 2 + Y 2 + Z 2 − 1| <
= | − Z − 1| = |Z + 1|
と同値である.
点 P(X, Y, Z) は S の外部にあるので X 2 + Y 2 + Z 2 − 1 > 0
したがってこの不等式は
2
2
2
X2 + Y 2 + Z2 − 1 <
= Z + 1, また Z + 1 <
= −(X + Y + Z − 1)
つまり S の外部で,かつ
(
)2
1
9
<
X2 + Y 2 + Z −
=4
2
(
)2
1
1
2
2
>
···⃝
または
,
X
+
Y
+
Z
+
1
=4
2
···⃝
2
である.ところが S と球体⃝
2 の表面球とは半径の差が中心間の距離に等しいので⃝
2 が S に内接
し, S の外部で⃝
2 を満たす点は接点 (0, 0, 1) 以外にない.
したがって V は⃝
1 を満たし,かつ S の外部となる点の集合である.
その体積は
4
π
3
( )3
3
4
19π
19 · 3.15
− π13 =
<
< 10
2
3
6
6
1.4.4
y′ =
(
t2
である. C 上の点 Q t, 2
t +1
直線 PQ の傾きは
2x
(x2 + 1)2
)
をとる.
t2
−a
t2 + 1
t
17
であるから,Q の満たすべき条件は
t2
−a
2t
t2 + 1
· 2
= −1
t
(t + 1)2
である.これを満たす 0 でない t が存在すればよい. t ̸= 0 のもとで整理すると
a=
となる.ここで f (t) =
t2
(t2 + 1)2
+
t2 + 1
2
t2
(t2 + 1)2
+
とおく.
t2 + 1
2
f ′ (t) =
(t2
2t
2t{1 + (t2 + 1)3 }
+ 2t(t2 + 1) =
2
+ 1)
(t2 + 1)2
ゆえに f (t) は t = 0 で極小かつ最小で f (0) =
1
2
また lim f (t) = +∞ なので
t→∞
y = f (t) と y = a が t ̸= 0 である共有点を持つための条件は
1
2
a>
1.4.5
Qk (0, 0, zk ) とおく.条件から
( )2 (
)2
k
k
+ 1−
+ zk 2 = 1
n
n
一方
(
)
(
)
1 k
k+1
k+1
k 1
= 1−
−
1−
=
2 n
n
n
n 2n
△OPk Pk+1
√
∴
1 1
Vk = ·
3 2n
∴
√
√
( )2 (
)2
( )2
2 1 k
k
k
k
1−
− 1−
=
·
−
n
n
6 n n
n
√
∫
n−1
∑
2 1√
lim
Vk =
x − x2 dx
n→∞
6 0
k=0
ここで
∫
0
1
∫
√
x − x2 dx =
1
√
0
(
)2
1
1
− x−
dx
4
2
1
1
であるから, x − = sin θ とおく.
2
2
√
(
)2
1
1
1
1
π
π
− x−
= cos θ , dx = cos θdθ であり,積分域が − → になるので
4
2
2
2
2
2
√
(
)2
∫ 1
∫ π2
1
1
1
− x−
cos2 θ dθ
dx =
π 4
4
2
0
−2
[
] π2
∫ π2
1
1
π
1
=
θ + sin 2θ
=
(1 + cos 2θ) dθ =
8 − π2
8
2
8
π
−
2
18
√
2
ゆえに求める値は
π
48
※ 1 小三角形の面積は △OP0 Pn の n 等分として求めてもよい.
1
※ 2 積分計算は,積分値が半径 の半円の面積であるから,それで代用することも可能である.
2
が,このような基本的な問題においては「円の面積が πr2 になる」ことまで含めて,しっかりと
置換積分で計算する方がよい.
1.4.6
(1)
元···
{1, 2, 3, 4, 5, 6, 7, 8}
1 回 · · · {5, 1, 6, 2, 7, 3, 8, 4}
2 回 · · · {7, 5, 3, 1, 8, 6, 4, 2}
3 回 · · · {8, 7, 6, 5, 4, 3, 2, 1}
(2) 1 <
=k<
= N のとき,1 回のシャッフルでこの範囲の各数の前に数が1つずつおかれる.ゆえ
に f (k) = 2k .
N +1 <
=k<
= 2N のとき.シャッフルを,この範囲の数をそのまま前に持ってくる段階と,そ
れぞれの数の後ろに 1 <
=k<
= N にあった数を置く操作に分ける.前半の操作でこの範囲の
数 k はでまず左へ N 動かされ,k − N の位置に来る.その上で各数の後ろに数が1つずつ
おかれることになる.ゆえに f (k) = 2(k − N ) − 1 = 2k − (2N + 1)
いずれも f (k) − 2k は 2N + 1 の倍数である.
(3) 整数 a と b を 2N + 1 でわった余りが等しいとき
a≡b
と書くことにする.
a ≡ b, c ≡ d
とする.
このとき
a = (2N + 1)qa + r1 , b = (2N + 1)qb + r1
c = (2N + 1)qc + r2 , d = (2N + 1)qd + r2
とおくと
a + c = (2N + 1)(qa + qc ) + (r1 + r2 )
b + d = (2N + 1)(qb + qd ) + (r1 + r2 )
ac = (2N + 1)2 qa qc + (2N + 1)(qa r2 + qc r1 ) + r1 r2
bd = (2N + 1)2 qb qd + (2N + 1)(qb r2 + qd r1 ) + r1 r2
であるから
a+c≡b+d
ac ≡ bd
19
が成り立つ.ゆえに関係 ≡ は和・差・積に関して,数の演算と同様にできる.
したがって (2) は 1, · · · , 2N の各 k に対して f (k) ≡ 2k を意味している.
f m (k) で m 回のシャッフルで数 k の現れる位置を表す.
f m (k) ≡ 2m k
···⃝
1
を数学的帰納法で示す. m = 1 のときは上の考察から成立.
m − 1 のときの成立するとする.つまり f m−1 (k) ≡ 2m−1 k .
2f m−1 (k) ≡ 2m k
m = 1 のときの成立を k = f m−1 (k) で用いると
f (f m−1 (k)) ≡ 2f m−1 (k)
f m (k) = f (f m−1 (k)) だから,あわせて m のときも⃝
1 が成立する.
ゆえに正整数 m に対して⃝
1 が成立する.
N = 2n−1 で m = 2n でこの結果を用いる.
f 2n (k) ≡ 22n k = (2n )2 k = (2N )2 k
ところが 2N ≡ −1 であるから
(2N )2 k ≡ (−1)2 k = k
∴
f 2n (k) ≡ k
···⃝
2
1, 2, 3, · · · , 2N はちょうど 2N + 1 で割った余りから 0 を除いたものである.
したがって⃝
2 は,数列 {1, 2, 3, · · · , 2N } の各数が 2n 回のシャッフルで,元の位置に戻る
こと,つまり数列 {1, 2, 3, · · · , 2N } が {1, 2, 3, · · · , 2N } に戻ることを示している.
1.5
阪大前期理系問題
出題者が具体的な数学的現象を提示し,受験生はそれを正確に把握して解析し論証し数値を求め
る,という数学の典型的な問題群である.
提示された数学的現象も比較的自然なもので,作為を感じさせず親しみやすい.ひらめかなけれ
ば解けない,という問題は一つもない.適切に文字でおき,系統立てて遂行する力を試す問題はか
りである.大学として数学にどのような力を求めてるかが明確な出題である.
小問がていねいすぎる.時間制限のあるなかではやむを得ないが,高校生が自分で勉強するとき
は,2 番のように小問の中で設定がされているものははずせないが,できるだけ途中の小問ははず
して直接最後の問題を考えるようにするとよい.分析力や構想力が身につく.
20
1.5.1
実数を係数とする 3 次方程式
x3 + ax2 + bx + c = 0
が異なる 3 つの実数解をもつとする.このとき a > 0, b > 0 ならば,少なくとも 2 つの実数解は
負であることを示せ.
1.5.2
1
を考える. a, b, c, d を d < c < 0 < b < a をみたす数とし,曲線 C
x
上の 4 点 P, Q, R, S をそれぞれ x 座標が a, b, c, d であるような点としたとき,四角形 PQSR
が長方形になっているとする.
平面上に双曲線 C : y =
(1) b, c, d を a を用いて表せ.
(2) 線分 PR と x 軸との交点を T ,線分 QS と y 軸との交点を U とするとき,線分 TU と曲
線 C が共通点をもたないような a の値の範囲を求めよ.
(3) a が (2) の範囲にあるとき,3 線分 PT, TU, UQ と曲線 C で囲まれた部分の面積 S(a) を
求めよ.
(4) a が (2) の範囲を動くとき,S(a) の増減を調べその最大値を求めよ.
1.5.3
α を |α| = 1 であるような複素数とし,複素数の列 {zn } を
z1 = 1, z2 =
α4
zn
α2 z¯n−2
,
=
(n = 3, 4, 5, · · ·)
2 zn−1
4 z¯n−1
で定める ただし, z¯n は複素数 zn の共役な複素数とする.
(1) 各 n に対し zn を求めよ.
√
1
3
(2) zn の実部と虚部をそれぞれ xn , yn とし,α = − + i
とおくとき,無限級数の和
2
2
∞
∑
xk ,
k=1
∞
∑
yk
k=1
をそれぞれ求めよ.
1.5.4
nをn>
=n
=k<
= 7 をみたす整数とし,1 つのさいころを投げる試行を n 回くり返す.このとき, 2 <
をみたす整 k に対し,「 n 回の試行のうち,同じ目が出るどの 2 つの試行も k 以上離れている」と
いう事象が起こる確率を pk と表す.ただし, i 番目の試行と j 番目の試行について,この 2 つの
試行は |i − j| だけ離れているということにする.
21
(1) p2 の値を求めよ.
(2) k >
= 3 のとき pk の値を求めよ.
(3) 「 n 回の試行において,同じ目が続くことはなく,しかも同じ目が出る試行の組でちょうど
2 だけ離れたものが少なくとも 1 組存在する」という事象が起こる確率を求めよ.
1.5.5
平面上に原点 O を中心とする半径 1 の円 C1 と点 P(0, sin α) を中心とする半径 1 の円 C2 があ
π
る.ただし 0 < α < とする.円 C2 と x 軸との交点を A, B とし,A, B を通り y 軸と平行な直
2
線をそれぞれ lA , lB とする.2 直線 lA , lB ではさまれた領域の部分で,円 C1 の外部で円 C2 の
内部であるものを D1 ,円 C2 の外部で円 C1 の内部であるものを D2 とする.いま, D1 , D2 を
それぞれ x 軸のまわりに 1 回転させてできる回転体の体積を V1 (α), V2 (α) とする.
(1) V1 (α), V1 (α) − V2 (α) をそれぞれ α を用いて表せ.
(2) α が 0 < α <
1.6
π
の範囲を動くとき,V1 (α) − V2 (α) の最大値を求めよ.
2
阪大前期理系解答
1.6.1
f (x) = x3 + ax2 + bx + c とおく. f ′ (x) = 3x2 + 2ax + b であるから,a > 0, b > 0 なら x >
=0
> 0 で 単調増加である.
で f ′ (x) > 0.つまり f (x) は x =
したがって x >
= 0 で f (x) = 0 となる x は多くても1つであり,f (x) = 0 が異なる 3 つの実数
解をもつので,重解もない.
ゆえに,x >
= 0 となる解は多くても 1 つ,つまり,負の実数解は少なくとも 2 つある.
※解と係数の関係からも論証できる.
1.6.2
(1) 四角形 PQSR が長方形になっているので
−→ −→ −→ −→
PQ = RS, PQ · PR = 0
これから
(
) (
) (
) (
)
1 1
1 1
1 1
1 1
b − a, −
= d − c, −
, b − a, −
· c − a, −
=0
b a
d c
b a
c a
つまり
)
(
)
(
)
(
)
(
1
1
1
1
= (d − c) 1, −
, (b − a) 1, −
· (c − a) 1, −
=0
(b − a) 1, −
ab
cd
ab
ac
すなわち
a − b = c − d, ab = cd, 1 +
22
1
=0
a2 bc
第 1 式と第 2 式から ab = c(c − a + b),つまり (c − a)(c + b) = 0 . c − a ̸= 0 より c − −b
.ゆえにまた第 2 式より d = −a である.さらに,第 3 式から a2 b2 = 1 .ともに正なので
1
b= .
a
1
1
∴ b = , c = − , d = −a
a
a
y
Q
U
P
O
2
x
S
R
T
(2)
(1) の結果と 0 < b < a より 1 < a .さらに (1) から長方形は y = x に関して対称な位置に
1
あり,直線 PR の傾きは 1 である.ゆえに直線 PR の式は y = x − a + .この式で y = 0
a
として 点 T の x 座標を求める.
(
)
1
∴ T a− , 0
a
1
である.
a
対称性から,線分 TU が移動して最初に C と接する接点は (1, 1) である.C の (1, 1) で
の接線は x + y = 2 .
線分 TU は傾き −1 なのでその式は x + y = a −
線分 TU と曲線 C が共通点をもたないような a の値の範囲は
a−
1
<2
a
1 < a の下でこれを解くと
1<a<1+
√
2
(3) 四角形 PQUT の面積から, C と線分 PQ で囲まれた部分を引けばよい.
PQ2 =
(
)2 (
)2
)2
(
( )2
1
1
1
1
a−
−a =2 a−
+
, PT2 = 2
a
a
a
a
であるから
)
(
)
(
1
1
1
· =2 1− 2
PQUT = 2 a −
a
a
a
1
である.
a
(
) ∫ a(
)
1
1
1
2 1− 2 −
−x + a + −
1
a
a x
a
つぎに線分 PQ の式は y = −(x − a) +
S(a)
=
23
(
)
[
(
)
]a
x2
1
−
+ a+
x − log x
2
a
1
=
1
2 1− 2
a
=
a2
3
2−
− 2 + 2 log a
2
2a
a
(4)
S ′ (a) = −a +
3
(a2 − 3)(a2 + 1)
2
+ 3 =−
a a
a3
√
√
√
√
1 < 3 < 1 + 2 であるから,a が 1 < a < 1 + 2 の範囲を動くとき,S(a) は 1 < a < 3
√
√
√
で増加,a = 3 で極大かつ最大, 3 < a < 1 + 2 で減少する.
√
最大値 S( 3) = log 3
1.6.3
(1) |α| = 1 なので α
¯=
1
である.
α
∴
同様に z4 =
α2 z¯1 z2
α10
=
4 z¯2
4
z3 =
α18
.
8
これから α のべき部分の指数の数列から階差をとって推測する.その結果 zn は
α(n−1)(n+2)
2n−1
zn =
と推測される.これを数学的帰納法で示す.
n = 1, · · · 4 では成立. n − 1, n − 1 で成立とする.このとき
α2
zn−1
α2 α−(n−3)n
αn +n−2
2(n−2)(n+1)
· z¯n−2 ·
=
·
·
α
=
4
z¯n−1
4
2n−3
2n−1
2
zn =
となり n で成立する.ゆえに
α(n−1)(n+2)
2n−1
zn =
(2) α = cos
2π
2π
+ i sin
なので α3 = 1 .
3
3
一方,自然数 N に対して
N
N
N
∑ ∑
∑
1
<
<1
z
|z
|
=
k =
k
2k−1
k=1
ゆえに
∞
∑
k=1
k=1
zk は收束する.
k=1
∴
∞
∑
k=1
zk =
∞
∑
k=1
24
(
xk + i
∞
∑
k=1
)
yk
(n + 3)2 + (n + 3) − 2 − (n2 + n − 2) = 6n + 12 なので αn
返す.
∞
∑
zk
2
+n−2
は 3 回毎に同じ値をくり
∞
∑
α(k−1)(k+2)
=
k=1
2k−1
k=1
∞
∑
=
1
23j−3
+ α4
∞
∑
1
+ α10
23j−2
j=1
j=1
(
)(
)
α α
1
=
1+ +
2
4
1 − 18
√
)
(
8
5 3 3
3
=
i
1+ α = +
7
4
7
7
∞
∑
∴
k=1
5
xk = ,
7
∞
∑
k=1
∞
∑
j=1
1
23j−1
√
3 3
yk =
7
1.6.4
(1) 最初は任意で,以後直前に出た目と異なる目が出続ける事象である.
p2 = 1 ·
∴
( )n−1 ( )n−1
5
5
=
6
6
(2) さいころの目は 6 までなので k >
= 7 のとき pk = 0
2<
=k<
= 6 のとき.(1) と同様に考え,手前直近の k − 1 個の範囲内にある目とは異なる目が
出続ける事象である.手前直近の k − 1 個の範囲内にある目の個数は 1 回目から k 回目の試
行では 0, 1, · · · , k − 1 個で,その後はつねに k − 1 個である.それら以外の目が出ればよい.
∴
pk =
まとめて
6
6−k+1
··· ·
·
6
6
(
6−k+1
6
)n−k
=
6
6−k+2
··· ·
·
6
6


 0
(
)n−k+1
pk =
6
6−k+2
6−k+1

·
 ··· ·
6
6
6
(
6−k+1
6
)n−k+1
(k >
= 7)
(2 <
=k<
= 6)
(3) 「 n 回の試行において,同じ目が続くことはなく,しかも同じ目が出る試行の組でちょうど
2 だけ離れたものが少なくとも 1 組存在する」という事象は,いいかえれば「 n 回の試行の
うち,同じ目が出るどの 2 つの試行も 2 以上離れている」事象のうちで「 n 回の試行のう
ち,同じ目が出るどの 2 つの試行も 3 以上離れている」ことがない事象である.
ゆえにその確率は
( )n−1
( )n−2
5
5 4
p2 − p3 =
−
6
6 6
25
1.6.5
√
(1) 直線 lA , lB の方程式は x = ± 1 − sin2 α = ± cos α である.
√
円 C1 の方程式を y について解くと y = ± 1 − x2 であり.円 C2 の方程式を y について
√
解くと y = ± 1 − x2 + sin α である.
2 つの領域 D1 , D2 はそれぞれ
√
√
1 − x2 <
y<
1 − x2 + sin α
=
=
√
√
2
D2 : − 1 − x 2 <
= y− <
= 1 − x + sin α
D1 :
と表される.
したがって
∫
=
− cos α
∫ cos α
2π
∫ cos0α
V2 (α) =
=
cos α
{π(
V1 (α) =
√
{2 1 − x2 sin α + sin2 α} dx
√
{π(1 − x2 ) − π(− 1 − x2 + sin α)2 } dx
− cos α
∫ cos α
2π
√
1 − x2 + sin α)2 − π(1 − x2 )} dx
√
{2 1 − x2 sin α − sin2 α} dx
0
ここで x = cos θ の置換を行うと,
∫ cos α √
∫
2
1 − x dx = −
∫
=
=
0
α
sin2 θ dθ
π
2
π
2
1 − cos 2θ
dθ
2
α
(
)
1 π
1
− α + sin 2α
2 2
4
この結果を代入して整理する.
(
V1 (α) = 2π
)
π
− α + 4π sin2 α cos α
2
また
V1 (α) − V2 (α) = 4π sin2 α cos α
(2)
V1′ (α) − V2′ (α) = 2π(2 cos 2α sin α + sin 2α cos α)
(1)
= 8π sin α(2 − 3 sin α)
2
√
したがって sin α =
大値は
(2)
√
2
となる α で極大かつ最大になる.このとき cos α =
3
√
2 1
8 3
4π · · √ =
π
3
9
3
26
1
なので,最
3
1.7
神大前期理系問題
問題はいずれも高校数学の基本的な概念を問うものである.それが工夫された具体的対象として
提示されている.その意味で良心的な「いい問題」である.
日頃からこのような問題を適切な指導のもとで考えていくようにすれば,力がつく.ぜひ厳格な
論証を心がけつつ解いてほしい.
1.7.1
0 でない複素数 z に対して, w = u + iv を
1
w=
2
(
)
1
z+
z
とするとき,次の問に答えよ。ただし,u, v は実数, i は虚数単位である。
(1) 複素数平面上で, z が単位円 |z| = 1 上を動くとき, w はどのような曲線を描くか.u, v が
みたす曲線の方程式を求め,その曲線を図示せよ.
(
)
π
(2) 複素数平面上で, z が実軸からの偏角 α 0 < α <
の半直線上を動くとき, w はどの
2
ような曲線を描くか.u, v がみたす曲線の方程式を求め,その曲線を図示せよ.
1.7.2
正の整数 n に対して,連立不等式
{
0<x<
=n
x<
y
= <
= 3x
の表す領域を Dn とする.次の問に答えよ.
(1) 領域 Dn 内にある格子点 P(x, y) の個数を Sn とする.Sn を n で表せ.ただし,格子点と
は x 座標と y 座標の両方が整数であるような点のことである.
−→
(2) 原点 O(0, 0) を始点とし,領域 Dn 内の格子点 P(x, y) を終点とする位置ぺクトル OP は,
ベクトル
−
→
−
→
v1 = (1, 1), →
v2 = (1, 2), −
v3 = (1, 3)
と 0 以上の整数 m1 , m2 , m3 を用いて
−→
−
→
→
OP = m1 →
v1 + m2 −
v2 + m 3 −
v3
と表せることを証明せよ.
27
1.7.3
正の実数 a, b に対して 2 つの曲線
C1 : ay 2 = x3 (x >
= 0, y >
= 0)
2
3
C2 : bx = y (x >
= 0, y >
= 0)
の原点 O 以外の交点を P とする. 次の問に答えよ.
(1) 交点 P の座標を求め,2 つの曲線 C1 , C2 の概形を描け.
(2) 2 つの曲線 C1 , C2 で囲まれる部分の面積を a と b で表せ, また,この面積が一定値 S であ
るように a, b が動くとき, 点 P の軌跡の方程式を求めよ.
1.7.4
関数 f (x) は任意の実数 x に対して定義されているとする.次の問に答えよ.
(1) f (x) が x = a において微分可能であることの定義を述べよ.
(2) 次の 2 つの命題のうち正しいものを選び,それが正しい理由を示せ.
(i) f (x) が x = a において連続ならば, 必ず,f (x) は x = a において微分可能である.
(ii) f (x) が x = a において連続であっても,f (x) は x = a において微分可能であるとは
限らない.
(3) 関数 f (x) = cos x が x = a において微分可能であることを,(1) で答えた定義を用いて証明
せよ.
1.7.5
数字 1, 2, · · · , N の書かれたカードが 1 枚ずつ N 枚入っている箱から,元に戻さすに 1 枚ずつ
k 枚のカードを引く試行を考える.ここで,2 <
=k<
= N とする.引いたカードの順に書かれてい
る数字を x1 , x2 , · · · , xk とする.次の問に答えよ.
(1) x1 < x2 < · · · < xk ,すなわち, k 枚のカードを数字の小さい順に引く確率 p を求めよ.
(2) i は整数で 2 <
= k をみたすとする.
=i<
{
x1 < x2 < · · · < xi−1
xi−1 > xi
である確率,すなわち k 枚のカードのうち i − 1 枚目までは小さい順にカードを引き i 枚目
に初めて i − 1 枚目よりも数字の小さいカードを引く確率 qi を求めよ.
(3) N は 5 以上の整数で,k = 5 とする. 2 <
= 5 をみたす各整数 i について上の (2) の事象
=i<
が起こるとき,得点 i 点が与えられるとする.それ以外のときの得点は 0 点とする.このと
き,得点の期待値を求めよ.
28
1.8
神大前期理系解答
1.8.1
(1) |z| = 1 より z z¯ = 1 ,つまり
1
= z¯ .
z
Re(z),Im(z) で,それぞれ z の実部,虚部を表す.このとき,
w=
1
(z + z¯) = Re(z)
2
1 = Re(z)2 + Im(z)2 より −1 <
= Re(z) <
=1
∴
v = 0 (−1 <
=u<
= 1)
図 1.
(2) z は
z = r(cos α + i sin α) (r > 0)
とおける.
{
}
1
r(cos α + i sin α) + (cos α − i sin α)
r
(
)
(
)
1
1
1
1
r+
cos α, v =
r−
sin α
u=
2
r
2
r
w=
∴
0<α<
1
2
π
,および r > 0 から u > 0 .そして
2
)2 (
)2
v
u
−
cos α
sin α
(
)2
(
)2
1
1
1
1
r+
−
r−
=1
4
r
4
r
(
=
u
v
±
= 0 ,つまり v = ±(tan α)u を漸近線とする
cos α sin α
双曲線の u > 0 の部分である.図 2.
cos α > 0, sin α > 0 であるから
v
v
v = −(tan α)u
O
-1 O
1
u
u
v = (tan α)u
図1
図2
29
1.8.2
(1) 0 < k <
= n に対して直線 x = k の上にある領域 Dn 内の格子点 P(x, y) の個数は 3k − k + 1 =
2k + 1 個.
n
∑
∴ Sn =
(2k + 1) = n(n + 1) + n = n2 + 2n
k=1
−→
(2) 位置ぺクトル OP が,
−→
−
→
→
OP = m1 →
v1 + m2 −
v2 + m 3 −
v3
と表せることは,領域 Dn 内の格子点 P(x, y) に対し,
{
m1 + m2 + m3 = x
m1 + 2m2 + 3m3 = y
となる 0 以上の整数 m1 , m2 , m3 が存在することと同値である.
この 2 式は
{
m1 + m2 + m3 = x
m2 + 2m3 = y − x
と同値である.これから
m1 = 2x − y + m3
m2 = −x + y − 2m3
つまり m1 = 2x − y + m3 >
= 0, m2 = −x + y − 2m3 >
= 0 となる m3 >
= 0 が存在すればよい.
これから
−x+y
>
= m3 >
= −2x + y
2
したがって
(i) −2x + y <
= 0 のとき.m3 = 0 にとればよく,このとき
(m1 , m2 , m3 ) = (2x − y, −x + y, 0)
は確かに条件を満たす.
(ii) −2x + y >
= 1 のとき.m3 = −2x + y にとればよく,このとき
(m1 , m2 , m3 ) = (0, 3x − y, −2x + y)
は確かに条件を満たす.
1.8.3
(1)
C1 : y = a− 2 x 2
1
2
C2 : y = b 3 x 3
1
3
であるから
3 −1 1
a 2 x2
2
2 1 1
: y ′ = b 3 x− 3
3
C1 : y ′ =
C2
30
3 −1 −1
a 2x 2 > 0
4
2 1 4
, y ′′ = − b 3 x− 3 < 0
9
, y ′′ =
C1 は下に凸,C2 は上に凸で原点以外の交点は
a− 2 x 2 = b 3 x 3
3
1
3
2
2
2
3
より, x = a 5 b 5 , y = a 5 b 5 .
y
2
1
C1
C2
3
a5 b5
O
3
x
2
a5 b5
(2) C1 , C2 で囲まれる部分の面積を S とおく.
∫
3
2
a5 b5
(
S=
1
3
2
3
− 12
b x −a
x
3
2
)
0
[
3 1 5
2 1 5
dx =
b 3 x 3 − a− 2 x 2
5
5
]a 35 b 25
=
0
S が一定のとき,x5 = a3 b2 = (5S)2 a, y 5 = a2 b3 = (5S)2 b となり,
x5 y 5 = (5S)4 ab = (5S)5 (x > 0, y > 0)
つまり
xy = 5S (x > 0, y > 0)
これが点 P の軌跡の方程式である.
1.8.4
(1) f (x) が x = a において微分可能であるとは
lim
x→a
f (x) − f (a)
x−a
が存在することである.
(2) ii が正しい.なぜなら f (x) = |x − a| は

 lim (x − a) = 0
x→a
lim f (x) =
x→a
 lim (−x + a) = 0
x→a
(x > a)
(x < a)
となり x = a で連続である.しかし


 lim x − a − 0 = 1
f (x) − f (a)  x→a x − a
lim
=
−x+a−0
x→a

x−a

= −1
 lim
x→a
x−a
より lim
x→a
f (x) − f (a)
が存在しないので微分可能ではない.
x−a
31
(x > a)
(x < a)
1
ab
5
(3) 関数 f (x) = cos x が x = a において連続であること,および lim
x→0
f (x) − f (a)
x→a
x−a
lim
sin x
= 1 を用いて示す.
x
cos x − cos a
x→a
x−a
)
(
)
(
x−a
x+a
sin
− 2 sin
2
2
= lim
x→a
x−a
)
(

x−a 



(
)
sin


x+a
2
= − sin a
= lim − sin
·
x→a 
x−a

2




2
=
lim
ゆえに, cos x は確かに x = a で微分可能である.
1.8.5
(1) 全事象の総数は N Pk である.それに対して x1 < x2 < · · · < xk となる事象の各々は,異
なる k 個の数の組 {x1 , x2 , · · · , xk } の各々と一対一に対応している.すなわち, N Ck 通
りある.
∴
(2) 同様に
{
p=
N Ck
N Pk
=
1
k!
x1 < x2 < · · · < xi−1
xi−1 > xi
となる事象の総数を求める.
異なる i 枚のカード y1 , y2 , · · · , yi を選びその最大のものを xi−1 に,それ以外の i − 1 枚
中の 1 枚を xi とし,他は小さい順に x1 , x2 , · · · , xi−2 とすればよい.
残る k − i 枚は N − i から順に選んで並べればよい.ゆえに該当する事象の総数は
∴
qi =
N Ci
· (i − 1) · N −i Pk−i
N Ci
· (i − 1) · N −i Pk−i
i−1
=
i!
N Pk
(3) 求める得点の期待値を E とする.
E=
5
∑
i=2
1.9
1
8
i−1 ∑
=
=
i!
(i
−
2)!
3
i=2
5
i·
京大後期理系問題
後期の問題は,前期のような殺伐としたものではなく,いろいろ考えさせられる問題が提出され
ている.
3 番,4 番はそれぞれじっくりと考える値打ちのある問題である.1 番も問題の一般化が可能な
広がりのあるものだ.
32
1.9.1
1 から n ( n >
= 2) までの番号が,順番に 1 つずつ書かれた n 枚の札が袋に入っている.この袋
の中から札を 1 枚ずつ取り出し,つぎの (i),(ii) のルールに従って A または B の箱に入れる.
(i) 最初に取り出した札は A の箱に入れる.
(ii) 2 番目以降に取り出した札は,その番号がそれまでに取り出された札の番号のどれよりも大
きければ A の箱に入れ,そうでないときは B の箱に入れる.
n 枚の札すべてを取り出し,箱に入れ終わったとき,B の箱にちようど 1 枚の札が入っている確
率を求めよ.
1.9.2
y2
= 1 と円 (x − a)2 + y 2 = b (b > 0) が相異なる 4 点で交わるという.このとき点
4
(a, b) のとりうる範囲を図示せよ.
楕円 x2 +
1.9.3
各面が鋭角三角形からなる四面体 ABCD において 辺 AB と辺 CD は垂直ではないとする.こ
のとき辺 AB を含む平面 α に点 C,点 D から下ろした垂線の足をそれぞれ C′ , D′ とするとき,
4 点 A,B, C′ , D′ がすべて相異なり,しかも同一円周上にあるように α がとれることを示せ.
1.9.4
f (x) は xn の係数が 1 である x の n 次式である.相異なる n 個の有理数 q1 , q2 , · · · , qn に対
して f (q1 ), f (q2 ), · · · , f (qn ) がすべて有理数であれば, f (x) の係数はすべて有理数であること
を,数学的帰納法を用いて示せ.
1.9.5
数列 {an }, {bn } を
a1 = 3,b1 = 2
an+1 = an 2 + 2bn 2 , bn+1 = 2an bn (n >
= 1)
で定める.
(1) an 2 − 2bn 2 を求めよ.
(2) lim
n→∞
an
を求めよ.
bn
33
1.9.6
]
π π
で定義された関数 f (x) が
閉区間 − ,
2 2
(
)
∫ π2
π
π
<
f (x) +
sin(x − y)f (y) dy = x + 1 − <
x
2 = = 2
−π
2
[
を満たしている f (x) を求めよ.
1.10
京大後期理系解答
1.10.1
全事象は,番号のついた n 枚のカードの取り出し方の総数なので, n! 通りある.
n 枚の札すべてを取り出し,箱に入れ終わったとき,B の箱に番号 k (1 <
=k<
= n − 1) のカード
が 1 枚入っているとする.
このとき k を除く n − 1 枚はつねに後ほど大きくなければならず,つまり小さい方から順に取り
出していなければならない.k のカードはそのうち k + 1 のカード後より,n のカード後までのど
こかで取り出されている.よって最後に k 1 枚が B にあるような取り出し方は n − k 通りである.
したがって該当する事象の総数は
∑
ゆえに,求める確率は
k = 1n−1 n − k =
n(n − 1)
2
n(n − 1)
1
=
2n!
2(n − 2)!
1.10.2
x と y の連立方程式

2
 x2 + y = 1 · · · ⃝
1
4

(x − a)2 + y 2 = b · · · ⃝
2
が異なる四つの実数解 (の組) を持てばよい.⃝
1 から y 2 = 4(1 − x2 ).一つの x の値に対して実
数 y が二つ定まるのは,
−1 < x < 1
のときである.⃝
1 , ⃝
2 から y を消去して整理した
3x2 + 2ax − a2 + b − 4 = 0
···⃝
3
が x の二次方程式なので,求める条件は ⃝
3 が −1 < x < 1 に異なる二つの実数解を持つことと
同値である.
f (x) = 3x2 + 2ax − a2 + b − 4 とおき, D/4 = a2 − 3(−a2 + b − 4) とする.放物線 y = f (x)
a
の軸は x = − であるから,条件は
3
D > 0, f (±1) > 0, −1 < −
34
a
<1
3
これを整理して,

4


b < a2 + 4


3


−3 < a < 3


b > (a + 1)2




b > (a − 1)2
図は次のようになる.
b
16
4
-3
-1
O
a
3
1
境界は含まない
1.10.3
α
C
D
△ABC
D
α
D′
図1
AB
C′ C
D′
C
D
C′
AB
AB
α
図2
図3
3 つの図は線分 AB の方から見たものである.
はじめ α を図 1 のように △ABC と垂直であるようにおく.このとき △ABC が鋭角三角形なの
で,点 C′ は線分 AB 上にある.
̸
AC′ B = π,
AD′ B < π
̸
α を図 2 のように C′ , D′ が線分 AB に関して同じ側にあるように連続的に動かす.
そして図 3 のように △ABD と α が垂直であるところまで動かす.同様にして
̸
AC′ B < π,
̸
AD′ B = π
α の位置を例えば △ABC とのなす角 θ で表すと,C, D の α への正射影 C′ , D′ の位置は角 θ
の三角関数で表され,角 θ に関して連続である.したがって二つの角 ̸ AC′ B,
̸
AD′ B も (正確に
は三角関数の逆関数も連続な範囲にあるので) 角 θ に関して連続である.
したがって連続関数の中間値の定理によって, α が図の範囲を動くとき途中で
̸
AC′ B = ̸ AD′ B
35
となるときがある.このとき円周角が等しく四点は同一円周上にある.
また辺 AB と辺 CD が垂直ではないので C′ ̸= D′ である.したがって四点 A,B, C′ , D′ は
すべて異なっている.
※別解
α を動かす範囲をもう少し狹めてもできる.α を △ABC が α 上にあるところから △ABD が
α 上にあるようになるまで,上と同じく C′ , D′ が線分 AB に関して同じ側にあるように連続的に
動かす.
D
C
Q
P
D′
C′
B
A
R
A
B
図5
図4
△ABC が α 上にあるとき, D′ はどこにあるか.
ここで,△ABC の外接円をかき,△ABC の各頂点から各辺に垂直な直線を引くと,それら
π
は 2 本ずつ外接円周上で交わる.例えば AR と BR について,̸ CAR = ̸ CBR =
なので,
2
̸ ACB + ̸ ARB = π となり,四点 A, C, B, R は同一円周上にあるからである.他も同様.
D′ は六角形 AQCPBR の内部にあり,したがって △ABC の外接円の内部にある.
円弧 AD′ B は △ABC の外接円の内部に含まれる.
△ABD が α 上にあるようになったときは,逆に円弧 AC′ B が △ABD の外接円の内部に含ま
れる.
α の動きに対して二つの円は連続的に変化するから,途中で二つの円が一致するときがある.
つまり,四点が同一円周上にあるときが存在する.
1.10.4
n = 1 のとき. f (x) = x + b とおく.f (q1 ) = q1 + b で f (q1 ) が有理数なので, b = f (q1 ) − q1
も有理数である.
n = m のとき成立するとする.
n = m + 1 のとき. f (x) を xm+1 の係数が 1 である x の m + 1 次式とし,相異なる m + 1 個
の有理数 q1 , q2 , · · · , qm+1 に対し f (q1 ), f (q2 ), · · · , f (qm+1 ) がすべて有理数であるとする.
因数定理より
f (x) = (x − qm+1 )Q(x) + f (qm+1 )
···⃝
1
となる.ここで Q(x) は m 次式で xm の係数は両辺の係数を比較して 1 である.
q1 , q2 , · · · , qm+1 はすべて異なるので,q1 , q2 , · · · , qm に対し
Q(qi ) =
f (qi ) − f (qm+1 )
qi − qm+1
36
したがって Q(q1 ), · · · , Q(qm ) はすべて有理数である.
数学的帰納法の仮定から Q(x) は有理数係数の多項式である.ゆえに ⃝
1 から f (x) も有理数係
数の多項式である.
したがって題意が示された.
※この問題は数学的帰納法を使うことなく解ける.
g(x) = f (x) − xn は n − 1 次式である.g(q1 ), g(q2 ), · · · , g(qn ) もすべて有理数である.
n − 1 次式 g(x) に対して n 個の x に対する値が定まれば g(x) は一意に定まる.実際
G(x)
(x − q2 )(x − q3 ) · · · (x − qn )
(q1 − q2 )(q1 − q3 ) · · · (q1 − qn )
(x − q1 )(x − q3 ) · · · (x − qn )
+g(q2 )
(q2 − q1 )(q2 − q3 ) · · · (q2 − qn )
+···
(x − q1 )(x − q2 ) · · · (x − qn−1 )
+g(qn )
(qn − q1 )(qn − q2 ) · · · (qn − qn−1 )
=
g(q1 )
とおく.このとき G(q1 ) = g(q1 ), · · · , G(qn ) = g(qn ) なので恒等式の原理より G(x) = g(x) .し
たがって
f (x)
=
xn + g(x)
= xn + g(q1 )
(x − q1 )(x − q2 ) · · · (x − qn−1 )
(x − q2 )(x − q3 ) · · · (x − qn )
+ · · · + g(qn )
(q1 − q2 )(q1 − q3 ) · · · (q1 − qn )
(qn − q1 )(qn − q2 ) · · · (qn − qn−1 )
これから f (x) が有理数係数の n 次多項式であることが示された.
これが「ラグランジュの補間公式」の方法である.
1.10.5
(1)
an+1 2 − 2bn+1 2
= (an 2 + 2bn 2 )2 − 2(2an bn )2
= an 4 − 4an 2 bn 2 + 4bn 4 = (an 2 − 2bn 2 )2
∴
n−1
an 2 − 2bn 2 = (a1 2 − 2b1 2 )2
=1
(2) an , bn は作り方から正整数で,しかも単調に増加する.
自然数の単調増加列であるから
lim an = +∞, lim bn = +∞
n→∞
n→∞
したがって (1) から
an 2
1
= 2 + lim
=2
n→∞ bn 2
n→∞ bn 2
lim
∴
√
an
= 2
n→∞ bn
lim
37
1.10.6
∫
π
2
−π
2
∫
sin(x − y)f (y) dy = sin x
である.そこで
∫
π
2
−π
2
∫
cos yf (y) dy − cos x
∫
π
2
−π
2
cos yf (y) dy = c1 ,
π
2
sin yf (y) dy
−π
2
π
2
−π
2
sin yf (y) dy = c2
とおく.条件から
f (x) = −c1 sin x + c2 cos x + x + 1
したがって, sin x cos x, x cos x が奇関数であることに注意して
∫
c1
π
2
=
cos yf (y) dy
∫
−π
2
π
2
=
−π
2
∫
=
(−c1 sin y + c2 cos y + y + 1) cos y dy
π
2
(c2 cos2 y + cos y) dy
)
∫ π2 (
1 + cos 2y
2
c2
+ cos y dy
2
0
[ (
)
] π2
1
1
2 c2
y + sin 2y + sin y
2
4
0
π
c2 + 2
2
∫ π2
sin yf (y) dy
2
0
=
=
=
c2
=
∫
−π
2
π
2
=
−π
2
∫
(−c1 sin y + c2 cos y + y + 1) sin y dy
π
2
(−c1 sin2 y + y sin y) dy
)
∫ π2 (
∫ π2
π
1 − cos 2y
= 2
−c1
dy + 2 [−y cos y]02 + 2
cos y dy
2
0
0
)] π2
[
(
π
1
1
y − sin 2y
= 2 −c1
+ 2 [sin y]02
2
4
0
π
= − c1 + 2
2
=
2
0
これから
c1 =
∴
f (x) = −
8 − 4π
8 + 4π
, c2 =
2
4+π
4 + π2
8 + 4π
8 − 4π
sin x +
cos x + x + 1
4 + π2
4 + π2
38
2
分野別
3
数
3.1
3.1.1
問題
名大前期理系
関係式
xa = y b = z c = xyz
を満たす 1 とは異なる 3 つの正の実数の組 (x, y, z) が,少なくとも 1 組存在するような,正の整
数の組 (a, b, c) をすべて求めよ.ただし, a <
=b<
= c とする.
3.1.2
名大前期文系
√
n を自然数とするとき,3 つの数 a =
5
√
1
1 + − 1 ,b = 1 −
n
5
1−
1
1
,c =
の大きさを比
n
5n
較せよ.
3.1.3
九大前期理系
正の整数 a に対し, a の正の約数全体の和を f (a) で表す.ただし,1 および a 自身も約数とす
る.たとえば f (1) = 1 であり,a = 15 ならば 15 の正の約数は 1,3,5,15 なので f (15) = 24 となる.
次の問いに答えよ.
(1) a が正の奇数 b と正の整数 m を用いて a = 2m b と表されるとする.このとき
f (a) = (2m+l − 1)f (b)
が成り立つことを示せ.
(2) a が 2 以上の整数 p と正の整数 q を用いて a = pq と表されるとする.このとき
f (a) >
= (p + 1)q
が成りたつことを示せ.また,等号が成り立つのは,q = 1 かつ p が素数であるときに限る
ことを示せ.
(3) 正の偶数 a, b は,ある整数 m, n とある奇数 r, s を用いて a = 2m r, b = 2n s のように表す
ことができる.このとき a, b が
{
f (a) = 2b
f (b) = 2a
をみたせば,r, s は素数であり,かつ r = 2n+1 − 1, s = 2m+1 − 1 となることを示せ.
39
3.1.4
慶応医 4 番
設問 (1) から (5) に答えなさい.
4 で割ると余りが 1 になるような素数 p, p = 4k + 1,を 1 つとる.これに対し,等式
a2 + 4bc = p
(Q)
を満たす自然数 3 つの組 (a, b, c) の全体を考える.両辺の絶対値を比べればわかるように,この
ような自然数 3 つの組の可能性は有限通りしかありえない.
いま等式 (Q) を満たす自然数 3 つの組 (a, b, c) から新しく自然数 3 つの組を作る手続きを次の
(i),(ii),(iii) により定める:
(i) a < b − c ならば (a + 2c, c, b − a − c) を作る;
(ii) b − c < a < 2b ならば (2b − a, b, a − b + c) を作る;
(iii) a > 2b ならば (a − 2b, a − b + c, b) を作る.
(1) (a, b, c) が等式 (Q) を満たす自然数の組でさらに (i) の条件 a < b − c を満たすとする.こ
のとき,上の (i) より得られる (a + 2c, c, b − a − c) もまた等式 (Q) を満たすことを示しな
さい.
(2) 等式 (Q) を満たす自然数の組 (a, b, c) は a = b − c や a = 2b を満たすことはないことを示
しなさい.
(3) 等式 (Q) を満たす自然数の組 (a, b, c) の中には,上の手続きを施しても変化しないという
性質を持つものが存在する.p = 4k + 1 と表すとき,この性質を持つ (a, b, c) を k を用い
て具体的に与え,かっそれがただ 1 組しか存在しないことを示しなさい.
(4) 等式 (Q) を満たす自然数の組 (a, b, c) に対して上の手続きを 2 回繰返して施すとどうなるか,
結論を簡潔に説明しなさい.また,この観察をもとに等式 (Q) を満たす自然数 3 つの組の全
体の個数が偶数か奇数かを決定し,そう判断できる理由を述べなさい.ただし,等式 (Q) を
満たす自然数 3 つの細から上の手続きにより新しく作られた自然数 3 つの組は (i),(ii),(iii)
のどの場合でも再び等式 (Q) を満たすという事実についてはここでは証明なしに用いてよい.
(5) 素数 p = 4k + 1 をある 2っの自然数 a, b により
p = a2 + (2b)2
と表すことができることを示しなさい.
3.1.5
金沢前期理系
x > 0 に対して,2 次の行列 A(x), B を
( √
)
(
1 + 3x2
3x
2
√
A(x) =
, B = A(1) =
2
x
1 + 3x
1
3
)
2
と定める.
(1) x > 1 のとき, 0 < y < x であって A(x) = A(y)B を満たす実数 y が存在することを示せ.
(2) 行列 A(x) の各成分が自然数であるとする.このとき,A(x) = B n となる自然数 n が存在
することを示せ.
40
3.1.6
金沢後期理系
p, q は互いに素な整数とし, 1 < p < q とする.座標平面内の集合 L を
L = { (m, n) | m, n は整数で 0 <
= m < q − 1, 0 <
=n<p−1 }
とし, L の各元 A(m, n) に対し N (A) = mp + nq とおく.
(1) L の各元 A, B について,N (A) = N (B) ならば A = B であることを示せ.
(2) L の各元 A(m, n) に対し,L の元 A# (q − 2 − m, p − 2 − n) を対応させる.A# ̸= A を
示せ.
#
(3) N (A) <
= pq − (p + q) となるためには,N (A ) >
= pq − (p + q) であることが必要十分条件で
あることを示せ.
(4) N (A) <
= pq − (p + q) を満たす L の元 A の個数を求めよ.
41
3.2
3.2.1
解答
名大前期理系
条件をみたす正の実数 (x, y, z) が存在したとする.
xa = y b = z c = xyz = k
1
1
1
とおく.このとき x = k a , y = k b , z = k c でさらに xyz = k なので
1
1
1
xyz = k a + b + c = k
x, y, z は 1 とは異なり,a, b, c は正なので k > 0, k ̸= 1 .
したがって
1 1 1
+ + =1
a b
c
が必要である.
1 1 1
1
1
1
逆に + + = 1 なら,任意の 1 でない正の実数 k に対して x = k a , y = k b , z = k c とお
a
b
c
けば,この x, y, z は条件をみたす.
1 1 1
したがって + + = 1 が,x, y, z が存在するための必要十分条件である.
a b
c
1
1
1
a<
=b<
= c より a >
= b >
= c である.
∴
1=
1 1 1
3
+ + <
=
a b
c
a
これから a = 1, 2, 3 でなければならない.
a = 1 なら b, c は存在しない.
a = 2 のとき
1
1 1
2
= + <
2
b
c = b
これから b = 3, 4
b = 3 なら c = 6 ,b = 4 なら c = 4
a = 3 のとき
2
1 1
2
= + <
3
b
c = b
これから b = 3 ,このとき c = 3
ゆえに求める正の整数の組 (a, b, c) は
(3, 3, 3), (2, 4, 4), (2, 3, 6)
3.2.2
名大前期文系
√
√
1
n を自然数として an = 5 1 + − 1 ,bn = 1 −
n
√
1
a1 = 5 2 − 1 ,b1 = 1 ,c1 = である.
5
42
5
1−
1
1
,cn =
とおく.
n
5n
ここで
(
)4
1
+ 1 = 1.24 = 1.44 · 1.44 = 2.0736 > 2
5
なので
⇐⇒
(c1 + 1)5 > 2 ,
∴
c1 > a1
b1 > c1 > a1
よって bn > cn > an と推測される.
√
bn > cn
⇐⇒
1−
1
1
−
>0
n 5n
(
)5
1
1
>1−
1−
5n
n
( )2
( )3
( )4 ( )5
1
1
1
1
− 5 C3
+ 5 C4
−
>0
5 C2
5n
5n
5n
5n
⇐⇒
⇐⇒
これは明らかに成立
cn > an
1−
5
√
1
1
− 5 1+ +1>0
5n
n
(
)5
1
1
1+
>1+
5n
n
( )3
( )4 ( )5
( )2
1
1
1
1
+
C
+
C
+
>0
C
5 3
5 4
5 2
5n
5n
5n
5n
⇐⇒
⇐⇒
⇐⇒
これは明らかに成立
したがって bn > cn > an である.
3.2.3
九大前期理系
(1) b の約数を bi (i = 1, 2, · · · , l) とする.2 と b は互いに素なので a = 2m b の約数のすべては,
2j bi (j = 0, 1, · · · , m, i = 1, 2, · · · , l)
で与えられる.
∴
f (a)
=
j=m,
∑i=l
j=0, i=1

= 
m
∑
j
2 bi =

m
∑
j=0
2j  f (b) =
j=0
(
2
j
l
∑
)
bi
i=1
2m+1 − 1
f (b) = (2m+1 − 1)f (b)
2−1
(2) p が 2 以上の整数なので pq ̸= q である.したがって q と pq は a = pq の異なる約数である.
∴
f (a) >
= (p + 1)q
等号が成り立つのは,a = pq の約数が q と a のみのときである.1 とその数自身は必ず約
数になるので q = 1 で,かつ 1 とその数自身以外の約数がないので p は素数でなければなら
ない.
43
(3) (1) から
{
f (a) = (2m+1 − 1)f (r) = 2b = 2n+1 s
f (b) = (2n+1 − 1)f (s) = 2a = 2m+1 r
···⃝
1
ここで,2m+1 − 1 と 2n+1 は互いに素なので, s は 2m+1 − 1 を約数にもつ.r についも同様.
s = (2m+1 − 1)s′ ,
r = (2n+1 − 1)r′
とおける.このとき ⃝
1 から
f (r) = 2n+1 s′ ,
f (s) = 2m+1 r′
···⃝
2
一方 (2) から
n+1
f (r) >
− 1) + 1}r′ = 2n+1 r′ ,
= {(2
m+1
f (s) >
− 1) + 1}s′ = 2m+1 s′
= {(2
···⃝
3
である.
n+1 ′
2n+1 s′ >
r
=2
かつ
m+1 ′
2m+1 r′ >
s
=2
より r′ = s′ で⃝
3 が等号になる.
(2) より r′ = s′ = 1 なので⃝
2 から
r = 2n+1 − 1,
3.2.4
s = 2m+1 − 1
慶応医 4 番
(1)
(a + 2c)2 + 4c(b − a − c) = a2 + 4bc
(a, b, c) は等式 (Q) を満たすので a2 + 4bc = p である.ゆえに (a + 2c, c, b − a − c) もま
た等式 (Q) を満たす.
(2) a = b − c とする.
p = a2 + 4bc = (b − c)2 + 4bc = (b + c)2
b + c が自然数なので, p が素数であることに反する.
a = 2b とする.
p = a2 + 4bc = (2b)2 + 4bc = 4b(b + c)
b, b + c が自然数なので, p が素数であることに反する.
ゆえに,a = b − c や a = 2b を満たすことはない.
(3) 手続きのうち,(i) と (iii) は,必ず変化する.変化しないときは手続き (ii) で
2b − a = a, a − b + c = c
44
となるときにかぎる.これから a = b .このとき
p = a(a + 4c)
p は素数,かつ a + 4c > 1 なので a = 1 .
このとき
p = 1 + 4c = 4k + 1
から c = k
したがって題意をみたすものは (a, b, c) = (1, 1, k) であり,これ以外には存在しない.
(4) 等式 (Q) を満たす自然数の組 (a, b, c) に対して上の手続きを 1 回行ったものを (a′ , b′ , c′ ),
2 回行ったものを (a′′ , b′′ , c′′ ) と記す.これらは等式 (Q) を満たす.
(i) a < b − c ならば a′ = a + 2c, b′ = c, c′ = b − a − c.このときは a′ > 2b′ なので
a′′ = a′ − 2b′ = (a + 2c) − 2c = a, b′′ = a′ − b′ + c′ = b, c′′ = b′ = c
(ii) b − c < a < 2b ならば a′ = 2b − a, b′ = b, c′ = a − b + c.このときは b′ − c′ = 2b − a − c
なので b′ − c′ < a′ < 2b′ となる.ゆえに
a′′ = 2b′ − a′ = 2b − (2b − a) = a, b′′ = b′ = b, c′′ = a′ − b′ + c′ = c
(iii) a > 2b ならば a′ = a − 2b, b′ = a − b + c, c′ = b.このときは a′ < b′ − c′ なので
a′′ = a′ + 2c′ = a, b′′ = c′ = b, c′′ = b′ − a′ − c′ = (a − b + c) − (a − 2b) − b = c
したがって 2 回の操作で元の組に戻る.
したがって組の各要素はたがいにこの操作で入れ替わるものの組に分けることができる.
この操作で変わらないものはただ一つなので,その他は 2 つずつの組になる.
したがって等式 (Q) を満たす自然数 3 つの組の全体の個数は奇数である.
(5) 等式 (Q) は b と c に関して対称である.したがって組 (a, b, c) が等式 (Q) を満たせば,組
(a, c, b) も満たす.
(3) から等式 (Q) を満たす自然数 3 つの組は少なくとも一組は存在し,(4) からそのような組
の全体の個数は奇数である.
もしすべての組が b ̸= c なら,2 つずつが組になってそのような組の全体の個数は偶数になる.
したがって,そのような組のなかには b = c となるものが存在する.このとき
p = a2 + (2b)2
と表される.
45
3.2.5
金沢前期理系
(1) A(x) = A(y)B より各成分を比較して,
√
√
1 + 3x2 = 2 1 + 3y 2 + 3y
√
x = 2y + 1 + 3y 2
√
3x = 3 1 + 3y 2 + 6y
···⃝
1
···⃝
2
···⃝
3
⃝
2 が成り立っているとき,
1 + 3x2
√
= 1 + 3{4y 2 + 4y 1 + 3y 2 + 1 + 3y 2 }
√
√
= 9y 2 + 12y 1 + 3y 2 + 4(1 + 3y 2 ) = {2 1 + 3y 2 + 3y}2
よって, ⃝
2 を満たす正の x が存在すれば,それは ⃝
1 , ⃝
3 を満たす.
z
z = 2y +
√
z = f (y) = 2y + 1 + 3y 2 とおく. f (y) は
y に関する単調増加関数で f (0) = 1 .さらに
√
1 + 3y 2
f (y) > y である.
したがって x > 1 のとき,z = f (y) と z = x
z=y
x
は yz 平面で y 座標が 0 < y0 < x である点で
交わる.
1
つまり題意をみたす y がただ一つ存在する.
O y0
y
x
(2) A(x) = A(y)B のとき,
(
A(y) = A(x)B
−1
= A(x)
2
−3
−1
2
)
である.
よって A(x) の各成分が自然数なら,0 < y < x より行列 A(y) の各成分も自然数である.
y > 1 なら同じ操作をくり返す.今 A(yk ) = A(x)B −k とおく. yk > 0 であるがさらに
yk > 1 であるかぎり yk > yk+1 となる.
y1 > y2 > · · · > 0
が得られる.自然数の単調減少列なので ym−1 > 1, ym = 1 となる k = m が存在する.つ
まり A(ym ) = A(x)B −m で A(ym ) = A(1) = B より A(x) = B m+1 .
ゆえに n = m + 1 とおくと確かに A(x) = B n となる自然数 n が存在した.
[注]
A(x) =
でu=
( √
1 + 3x2
x
√
1 + 3x2
√
1 + 3x2 とおくと
u2 − 3x2 = 1
46
3x
)
となる.つまり A(x) で x が自然数のとき,その 1 − 1 成分と 1 − 2 成分は,ペル方程式
X 2 − 3Y 2 = 1
の自然数解になっている.
これは構造定理の別解になっている.
3.2.6
金沢後期理系
(1) A(k, l), B(m, n) とする.
N (A) = N (B) より kp + lq = mp + nq である.つまり
p(k − m) = q(n − l)
であるが, p と q が互いに素なので k − m が q の倍数でなければならない.ところが
0<
= k, m < q − 1 なので
−(q − 1) < k − m < q − 1
である.この範囲で q の倍数は 0 しかない.つまり k = m .
その結果 l = n となり,A = B であることが示された.
(2) A# = A とする.つまり
q − 2 − m = m, p − 2 − n = n
これから
q = 2m + 2, p = 2n + 2
となり, p と q は公約数 2 をもち互いに素であることに反する.ゆえに A# ̸= A である.
(3) 条件 N (A) <
= pq − (p + q) は
mp + nq <
= pq − (p + q)
である.他方,条件 N (A# ) >
= pq − (p + q) は
(q − 2 − m)p + (p − 2 − n)q >
= pq − (p + q)
である.ここで
⇐⇒
(q − 2 − m)p + (p − 2 − n)q >
= pq − (p + q)
pq − mp − nq − p − q >
=0
⇐⇒
pq − (p + q) >
= mp + nq
ゆえに 2 つの条件が同値であることが示され,題意が示された.
(4) (3) から N (A) = pq − (p + q) なら N (A# ) = pq − (p + q) となる.つまり等号が成立すると
N (A) = N (A# ) である.(1) から A = A# となるが,これは (2) の結果と矛盾する.ゆえに
N (A) <
= pq − (p + q) で等号は成立しない.
47
A(m, n) のとき
(A# )# = (q − 2 − (q − 2 − m), p − 2 − (p − 2 − n)) = (m, n) = A
なので,L の元 A と A# は 1:1 に対応する.(4) から N (A) <
= pq − (p + q) となる L の元 A
の個数は L の半分である.
L は明らかに (p − 1)(q − 1) 個からなるので,求める元の個数は
(p − 1)(q − 1)
2
式
4
4.1
4.1.1
問題
横国大後期理系
x の多項式 f (x) で, f (0) = 0 かつ f (1) = f (−1) を満たすものすべての集合を M とする.定
数 a, b, c, d に対し, g(x) = x2 + ax + b ,h(x) = x3 + cx + d が M の要素であるとき,次の問
いに答えよ.
(1) a, b, c, d の値を求めよ.
(2) M の任意の要素は,次の形の多項式をいくつか加え合わせたものであることを示せ.
k(g(x))m (h(x))n
ただし, k は定数, m, n は 0 以上の整数であり,(g(x))0 = 1, (h(x))0 = 1 とする.
4.1.2
京都府立医大
(1) 実数を係数とする任意の 4 次式は実数を係数とする 2 つの 2 次式の積であることを示せ.た
だし, n 次方程式が複素数の範囲で解をもつことは知られているとする.
(2) f (x) は次の実数を係数とする 4 次式とする.
f (x) = x4 + ax3 + bx2 + cx + d
このとき,次の条件を満たす実数を係数とする 2 次式 g(x) = px2 + qx + r が存在すること
を示せ.
4 次方程式 f (x) = g(x) は 4 つの相異なる実数解をもつ.
4.1.3
慶応理工
整数を係数とする多項式 f (x) について次のことを証明せよ.
(1) 任意の整数 m, n に対し f (m + n) − f (n) は m の倍数である.
(2) 任意の整数 k, n に対し f (n + f (n)k) − f (n) は f (n) の倍数である.
(3) 任意の自然数 n に対し f (n) が素数であるならば, f (x) は定数である.
48
4.1.4
岐阜後期理系
次の問に答えよ.
(1) 方程式 t3 − 3t = 1 は整数の解を持たないことを示せ.
(2) 整数 m と整数 n が n3 − 3m2 n − m3 = 0 を溝たしているとする.|m| > 1 ならば |m| と |n|
の最大公約数は 2 以上であることを示せ.
(3) 方程式 t3 − 3t = 1 は有理数の解を持たないことを示せ.
(4) a, b, c を有理数とする.これらが
a2 + 2b2 + 3c2 − ab − 2bc − 3ca
= b2 + 2c2 + 3a2 − bc − 2ca − 3ab
= c2 + 2a2 + 3b2 − ca − 2ab − 3bc
を満たすとき,a = b = c を示せ.
4.2
4.2.1
解答
横国大後期理系
(1) g(0) = b, g(1) = 1 + a + b, g(−1) = 1 − a + b なので a = b = 0 .
h(0) = d, h(1) = 1 + c + d, h(−1) = −1 − c + d なので c = −1, d = 0 .
(2) M の任意の要素 f (x) は f (0) = 0 より定数項は 0 なので,f (x) を偶数次数の項と奇数次数
の項に分けて
f (x) =
n
∑
m
∑
ak x2k +
k=1
bk x2k−1
k=1
とおく.条件 f (1) = f (−1) より
n
∑
ak +
k=1
つまり
m
∑
bk =
k=1
n
∑
k=1
m
∑
ak −
m
∑
bk
k=1
bk = 0
k=1
したがって b1 = −(b2 + b3 + · · · + bm ) である.
∴
m
∑
bk x2k−1 =
k=1
m
∑
bk (x2k−1 − x)
k=2
ここで
x2k−1 − x
= (x2k−1 − x2k−3 ) + (x2k−3 − x2k−5 ) + . . . + (x3 − x)
= x2k−4 (x3 − x) + x2k−6 (x3 − x) + . . . + (x3 − x)
= g(x)k−2 h(x) + g(x)k−3 h(x) + . . . + h(x)
f (x) は x2k と x2k−1 − x で構成されていて,各々が k(g(x))m (h(x))n で書き表されるので,
この形の多項式をいくつか加え合わせたものであることが示された.
49
[(2) の別解] M の要素の次数に関する数学的帰納法で示す.M の要素 f (x) をとる. f (x) = an xn + · · ·
と最高次数の係数を an とおく.
f (x) ∈ M の次数が 0 のとき. f (0) = 0 より f (x) = 0 ,つまり a0 = 0.定数 k を k = 0
にとれば成立.
f (x) ∈ M の次数が 1 のとき. f (x) = a1 x + a0 とおく.条件から a0 = a1 = 0 つまり集合
M に 1 次式は存在しない.
0, 2, · · · , n − 1 までの次数について成立するとし,次数 n のときに成立することを示す.
(i) n = 3j つまり n が 3 の倍数のとき.F (x) = f (x) − an h(x)j とおく. F (0) = f (0) −
an h(0)j = 0 ,F (1) = f (1) − an h(1)j ,F (−1) = f (−1) − an h(−1)j なので F (1) =
F (−1) も成立.ゆえに F (x) ∈ M で次数は n より小さいので, 数学的帰納法の仮定に
より F (x) は k(g(x))m (h(x))n の形の多項式の和.したがって f (x) = F (x) + an h(x)j
も k(g(x))m (h(x))n の形の多項式の和である.
(ii) n = 3j − 1 のとき. n = 3(j − 1) + 2 なので F (x) = f (x) − an g(x)h(x)j−1 とおく.同
様に F (x) ∈ M で次数は n より小さい.ゆえに (i) とおなじく f (x) は k(g(x))m (h(x))n
の形の多項式の和である.
(iii) n = 3j − 2 のとき. n = 1 ならすでに示されたので j >
= 2 とする.n = 3(j − 2) + 4
2
j−2
なので F (x) = f (x) − an g(x) h(x)
とおく.同様に F (x) ∈ M で次数は n より小さ
い.ゆえに (i) とおなじく f (x) は k(g(x))m (h(x))n の形の多項式の和である.
したがって, n のときも成立し,題意が示された.
4.2.2
京都府立医大
(1) F (x) = Ax4 + ax3 + bx2 + cx + d を実数を係数とする 4 次式とする.F (x) = 0 の解を α と
する. α が虚数の場合,α で α の共役複素数を表すと,F (α) = 0 より,
F (α)
=
Aα4 + aα3 + bα2 + cα + d
= A(α)4 + a(α)3 + b(α)2 + c(α) + d
= A(α)4 + a(α)3 + b(α)2 + c(α) + d
= F (α) = 0
したがって, α もまた F (x) = 0 の解である.
さて因数定理によって F (x) は
F (x) = (x − α)Q(x)
と因数分解され, Q(x) は 3 次式である.Q(x) = 0 の解を β とする.再び
Q(x) = (x − β)Q2 (x)
つまり F (x) = (x − α)(x − β)Q2 (x)
これをくり返せば, F (x) は 1 次式 4 つと定数の積になる. x4 の係数比較から
F (x) = A(x − α)(x − β)(x − γ)(x − δ)
50
と表される.
α, β, γ, δ が実数なら A(x − α)(x − β) ,(x − γ)(x − δ) はそれぞれ実数係数の 2 次式なの
で成立.
α が虚数のとき,上に見たように β, γ, δ のいずれかは α である.これを β とする.この
とき
(x − α)(x − α) = x2 − (α + α)x + αα
より,A(x − α)(x − α) は実数係数の 2 次式である.
(x − γ)(x − δ) は γ, δ がともに実数か,たがいに共役な虚数なのでやはり実数係数の 2 次式
である.
よって実数を係数とする任意の 4 次式は実数を係数とする 2 つの 2 次式の積であることが示
された.
(2) (1) から f (x) は実数を係数とする 2 つの 2 次式の積に分解される.それを
f (x) = (lx2 + mx + n)(sx2 + tx + u)
とする. ls = 1 なので
(lx2 +mx+n)(sx2 +tx+u) = (lsx2 +msx+ns)(slx2 +tlx+ul) = (x2 +msx+ns)(x2 +tlx+ul)
となる.よってはじめから x2 の係数が 1 の 2 つの 2 次式 h(x), k(x) で
f (x) = h(x)k(x)
と分解されるとしてよい.
十分大きな 2 つの実数 v, w を適当にとると
h(x) − v = 0, k(x) − w = 0
がともに相異なる 2 つずつの解をもつようにできる.つまり
(h(x) − v)(k(x) − w) = 0
は相異なる 4 つの実数解をもつ.この等式は
h(x)k(x) = vk(x) + wh(x) − vw
と変形される.したがって
g(x) = vk(x) + wh(x) − vw
とおけば,確かに g(x) = vk(x)+wh(x)−vw は実数係数の 2 次式で,4 次方程式 f (x) = g(x)
は 4 つの相異なる実数解をもつ.
51
4.2.3
慶応理工
(1) f (x) を N 次多項式とし, f (x) =
N
∑
aj xj とおく.ここで各 aj は整数である.
j=0
f (m + n) − f (n) =
N
∑
aj (m + n)j −
j=0
ここで,
(
(m + n) − n =
j
j
j
∑
i j−i
n
a j nj =
j=0
)
j Ci m
N
∑
−n =
j
i=0
j
∑
N
∑
aj {(m + n)j − nj }
j=0
(
j Ci m
i j−i
n
=m
i=1
j
∑
)
j Ci m
i−1 j−i
n
i=1
である.ゆえに f (m + n) − f (n) は m の倍数である.
(2) f (n) は整数なので,(1) で m = f (n)k とおくと f (n + f (n)k) − f (n) は m = f (n)k の倍数
である.したがって任意の整数 k に対し f (n + f (n)k) − f (n) は f (n) の倍数である.
(3) (2) から f (n + f (n)k) − f (n) は f (n) の倍数であるので,k によって定まる定数 M (k) を用
いて
f (n + f (n)k) − f (n) = M (k)f (n)
つまり
f (n + f (n)k) = {M (k) + 1}f (n)
と表せる.
任意の自然数 n に対し f (n) が素数なので f (n + f (n)k) も素数.
f (n) = 1 となる n が無数にあれば恒等式の原理から f (x) は定数.
f (n) = 0 となる n が無数にあれば恒等式の原理から f (x) は定数.
f (n) が 0 でも 1 でもない n が存在するとき,その n を固定する.
このときすべての k に対して {M (k) + 1} = 1.つまり
f (n + f (n)k) = f (n)
がすべての k について成立. f (n) ̸= 0 より f (x) = f (n) となる x が無数に存在する.
したがってこの場合も同様に f (x) は定数である.
4.2.4
岐阜後期理系
(1) t3 − 3t = t(t2 − 1) = 1 である.t が整数なら t = ±1 である.いずれも t3 − 3t = 1 を満たさ
ない.ゆえに整数の解はない.
(2) n3 − 3m2 n − m3 = 0 より,
n3
= 3n + m
m2
右辺は整数である.つまり n3 が m2 の倍数である.もし n と m が互いに素なら n3 と m2 も
互いに素である.|m| > 1 よりこのとき左辺は整数にはなり得ない.
したがって n と m が互いに素ではない.ゆえに,|m| > 1 ならば |m| と |n| の最大公約数は
2 以上である.
52
(3) 方程式 t3 − 3t = 1 が有理数の解を持つとし,それを t =
(
n
m
)3
(
−3
n
m
n
(m と n は互いに素) とする.
m
)
=1
より n3 − 3m2 n − m3 = 0.(2) より,m と n が互いに素なら |m| = 1.つまり解は整数であ
る.ところがこれは方程式 t3 − 3t = 1 には整数解が存在しないという (1) の結果と矛盾して
いる.
よって有理数の解も存在しない.
(4)
a2 + 2b2 + 3c2 − ab − 2bc − 3ca = k
···
⃝
1
b + 2c + 3a − bc − 2ca − 3ab = k
···
⃝
2
c2 + 2a2 + 3b2 − ca − 2ab − 3bc = k
···
⃝
3
2
2
2
とおく.(⃝
1 +⃝
2 +⃝
3 )÷ 3 より
2a2 + 2b2 + 2c2 − 2ab − 2bc − 2ca = k
···⃝
4
⃝
1 −⃝
4 より
−a2 + c2 + ab − ca = 0
∴
c2 − ca = a2 − ab
∴
b2 − bc = a2 − ab
同様に ⃝
2 −⃝
4 より
ここで
a2 − ab = h · · ·
⃝
5
b − bc = h · · ·
⃝
6
c − ca = h · · ·
⃝
7
2
2
とおく.
もし,b = 0 なら ⃝
6 より h = 0,⃝
5 より a = 0,⃝
7 より c = 0.つまり a = b = c が成立
する.
b ̸= 0 なら ⃝
6 より
b2 − h
= c.⃝
7 に代入して
b
(b2 − h)2
b2 − h
−
·a=h
b2
b
これから
b4 − 3b2 h + h2 − ab3 + abh = 0
53
···⃝
8
を得る.ここに h = a2 − ab を代入して整理すると,
0 =
b4 − 3b2 (a2 − ab) + (a2 − ab)2 − ab3 + ab(a2 − ab)
= a4 − ba3 − 3b2 a2 + 2b3 a + b4
=
(a − b)a3 − b2 (3a2 − 2ab − b2 )
=
(a − b)a3 − b2 (3a + b)(a − b)
=
(a − b)(a3 − 3ab2 − b3 )
ここで a − b ̸= 0 なら,
a3 − 3ab2 − b3 = 0
b ̸= 0 より
( )3
( )
a
a
−3
=1
b
b
a
という有理数が方程式 t3 − 3t = 1 の解となっているので (3) の結果と矛盾する.
b
よって a = b.このとき ⃝
5 から h = 0.b ̸= 0 なので ⃝
6 より b = c.
これは
∴
a=b=c
以上からつねに a = b = c が成立する.
図形
5
5.1
5.1.1
問題
北大前期理系
2 点 (1, 0, 0) , (0, 2, 0) を通る直線を l とし,中心が R(0, 0, 2) で半径が 1 の球面を C と
する.点 P が l 上にあり点 Q が C 上にあるとし,線分 PQ は直線 l と線分 RQ に垂直であると
する.
(1) 点 P の存在する範囲を求めよ.
(2) 線分 PQ の長さを最小にする点 P の座標を求めよ.
5.1.2
東北大前期理系
四面体 ABCD は各辺の長さが1の正四面体とする.
−→
−→
−→
−→
−→
−→
−→
(1) AP = lAB + mAC + nAD で与えられる点 P に対し |BP| = |CP| = |DP| が成り立つなら
−→
ば, l = m = n であることを示せ.また,このときの |BP| を l を用いて表せ.
(2) A, B, C, D のいずれとも異なる空間内の点 P と点 Q を,四面体 PBCD と QABC がとも
に正四面体になるようにとるとき, cos ̸ PBQ の値を求めよ.
54
5.1.3
九大前期理系
複素数平面上の原点を中心とする半径 1 の円 C に相異なる 3 点 z1 , z2 , z3 をとる.次の問いに
答えよ.
(1) w1 = z1 + z2 + z3 とおく.点 w1 は 3 点 z1 , z2 , z3 を頂点とする三角形の垂心になることを
示せ.ここで三角形の垂心とは,各頂点から対辺またはその延長線上に下ろした 3 本の垂線
の交点のことであり,これら 3 本の垂線は 1 点で交わることが知られている.
(2) w2 = −z1 z2 z3 とおく. w2 =
̸ z1 のとき,2 点 z2 , z3 を通る直線上に点 z1 から下ろした垂
線またはその延長線が円 C と交わる点は w2 であることを示せ.ここで z1 は z1 に共役な
複素数である.
(3) 2 点 z2 , z3 を通る直線とこの直線上に点 z1 から下ろした垂線との交点は,点 w1 と w2 を
結ぶ線分の中点であることを示せ.ただし, w1 = w2 のときは,w1 と w2 の中点は w1 と
解釈する.
5.1.4
名大前期文系
−→ −→
O を原点とする座標平面上の曲線 y = x2 上の 2 点 A,B に対し,OA · OB = t とおく.
(1) t のとりうる値の範囲を求めよ.
−→ −→ −→
(2) t = 2 のとき,OP = OA + OB となる点 P の軌跡を求めよ.
5.1.5
一橋
a は正の定数とする.点 (x, y) は条件 a|x| + |y| <
= a を満たす.
(1) y − (x + 1)2 の最小値を求めよ.
(2) y − (x + 1)2 の最大値を求めよ.
5.1.6
広大後期理
→ −→ →
−
→
−→ → −→ −
−
→
四面体 OABC において OA = −
a , OB = b , OC = −
c と表し,|→
a | = 1, | b | = 1, |−
c | = 1,
1
−
→
→
− −
−
→
−
→
→
→
−
a · b = , a · c = 0, b · c = p とする.
2
(1) p の取りうる値の範囲を求めよ.
−→
(2) 点 B を通る直線が,三角形 OAC を含む平面と垂直に交わる点を H とする.ベクトル BH を
−
→ −
−
→
a, b, →
c と p を用いて表せ.
(3) p を用いて,四面体 OABC の体積を表し,その体積が最大となる p の値を求めよ.
55
5.1.7
福井医大
放物線 y =
x2
上に相異なる 3 点 A(2a, a2 ), B(2b, b2 ), C(2c, c2 ) がある.このとき以下の問い
4
に答えよ.
(1) 3 点 A, B, C における法線が 1 点で交わるための必要十分条件は a + b + c = 0 であること
を証明せよ.
(2) (1) の条件をみたすとき,3 法線の交点を P とおく.さらに △ABC が直角三角形となるよう
に A, B, C が動くとき,P の軌跡を求めよ.
5.2
5.2.1
解答
北大前期理系
z
接平面
C
Q
2
α
x
l
P
y
(1) 線分 PQ が直線 l と垂直なので,線分 PQ は点 P を通り直線 l と垂直な平面上 α にある.
線分 PQ が RQ に垂直で,線分 RQ は球面 C の半径なので,線分 PQ は点 Q で球 C と
接する平面 (接平面) 上にある.
ゆえに線分 PQ は平面 α と,Q での接平面の交わりで定まる直線の一部である,
逆に直線 l と垂直な平面 α が球面 C と共有点をもつとする.α と C の共有部分は円また
は 1 点で,円の場合は点 P から円への接線を引き接点を Q とし,1 点の場合はその共有点
を Q とする.線分 PQ は点 Q での接平面上にあり,線分 PQ と線分 RQ は直交する.
ゆえに P の存在する範囲は,平面 α が C と共有点をもつ範囲として定まる.
球面 C の方程式は x2 + y 2 + (z − 2)2 = 1 である.平面 α は z 軸と平行なので α が球面 C
と接することと,球面 C の xy 平面への正射影の円と平面 α の xy 平面への正射影の直線が
接することとが同値である.
球面 C の xy 平面への正射影の円は x2 + y 2 = 1 である.平面 α の xy 平面への正射影の直
線は点 P を通り直線 l と直交する直線である.
56
xy 平面で l は x +
y
= 1 と書ける.よって点 P を (t, 2(1 − t)) とおく.平面 α の正射影
2
の直線を
(x − t) − 2(y − 2(1 − t)) = 0
とおく.これが円 x2 + y 2 = 1 と共有点をもてばよい.
|(0 − t) − 2(0 − 2(1 − t))|
<
√
=1
1+4
これを解いて
√
√
4+ 5
4− 5
<
=t<
=
5
5
点 P の存在する範囲は
y
x + = 1, z = 0
2
∴
(
√ )
√
4+ 5
4− 5
<
=x<
=
5
5
(2) 条件から
RQ2 + PQ2 = RP2
である.ゆえに
PQ2
= RP2 − RQ2
= t2 + {2(1 − t)}2 + 22 − 1 = 5t2 − 8t + 7
(
)2
4
= 5 t−
+1
5
t=
5.2.2
4
で最小になる.それを与える点 P は
5
(
)
4 2
, , 0
5 5
東北大前期理系
四面体 ABCD は各辺の長さが1の正四面体とする.
π
1
−→ −→ −→ −→ −→ −→
(1) AB · AC = AC · AD = AD · AB = 1 · 1 · cos = である.ゆえに
3
2
−→ 2
−→ −→ 2
|BP| = |BA + AP|
−→
−→
−→
= |(l − 1)AB + mAC + nAD|2
= (l − 1)2 + m2 + n2 + (l − 1)m + mn + n(l − 1)
−→
−→
他も同様である.したがって |BP| = |CP| より
(l − 1)2 + m2 + n2 + (l − 1)m + mn + n(l − 1) = l2 + (m − 1)2 + n2 + l(m − 1) + (m − 1)n + nl
これから l = m を得る.同様に m = n であるから l = m = n が示された.
−→
このとき |BP|2 = 6l2 − 4l + 1 なので
√
−→
|BP| = 6l2 − 4l + 1
57
−→
(2) 点 P は (1) の条件を満たし,かつ |BP| = 1 となる点である.ゆえに 6l2 − 4l + 1 = 1 より
2
2
l = 0,
. l = 0 は P = A のときだから, l = .つまり
3
3
−→ 2 −→ 2 −→ 2 −→
AP = AB + AC + AD
3
3
3
同様に
−→ 2 −→ 2 −→ 2 −→
DQ = DA + DB + DC
3
3
3
よって
1 −→ 2 −→ 2 −→
−→ −→ −→
BP = AP − AB = − AB + AC + AD
3
3
3
同様に
−→
−→ −→ 2 −→ 1 −→ 2 −→
BQ = DQ − DB = DA − DB + DC
3
3
3
2 −→ 1 −→ −→
2 −→ −→
= − AD − (AB − AD) + (AC − AD)
3
3
3
1 −→ 2 −→ −→
= − AB + AC − AD
3
3
これから
−→ −→
BP · BQ =
(
) (
)
1 −→ 2 −→ 2 −→
1 −→ 2 −→ −→
7
− AB + AC + AD · − AB + AC − AD = −
3
3
3
3
3
18
−→
−→
|BP| = |BQ| = 1 なので
∴
5.2.3
−→ −→
BP · BQ
7
cos ̸ PBQ = −→ −→ = −
18
|BP||BQ|
九大前期理系
(1) 3 点 z1 , z2 , z3 は半径 1 の円 C 上の点なので,
|zi | = 1 つまり
zi =
1
(i = 1, 2, 3)
zi
が成り立つ.
直線 z1 w1 と直線 z3 z2 が直交していることを示すためには,
w1 − z1
z2 − z3
が純虚数であることを示せばよい.ここで
w1 − z1
z2 − z3
=
=
z2 + z3
(z2 + z3 )(z2 − z3 )
=
z2 − z3
|z2 − z3 |2
2
z3 z2 − z3 z2
|z2 | + z3 z2 − z3 z2 − |z3 |2
=
2
|z2 − z3 |
|z2 − z3 |2
58
···⃝
1
z3
が実数なら原点と z2 , z3 が 1 直線上にあることになる.ゆえに実数ではなく,⃝
1
z2
は共役 2 虚数の差なので純虚数である.ゆえに直線 z1 w1 と直線 z3 z2 は直交している.
z3 z2 =
同様に直線 z2 w1 と直線 z1 z3 も直交しているので点 w1 は 3 点 z1 , z2 , z3 を頂点とする三
角形の垂心になる
(2) |w2 | = |z1 z2 z3 | = 1 より w2 は C 上の点である.
w2 − z1
− z1 z2 z3 − z1
=
z2 − z3
z2 − z3
1 1
− z1
− z1
− z1 − z1 z2 z3
z2 z3
=
=
1
1
z3 − z2
−
z2
z3
(
)
w2 − z1
w2 − z1
= −
=−
z2 − z3
z2 − z3
したがって確かに w2 は,2 点 z2 , z3 を通る直線上に点 z1 から下ろした垂線またはその延
長線が円 C と交わる点である.
(3) (1),(2) から z1 , w1 , w2 は同一直線上にあり,その直線は z2 , z3 を通る直線と直交して
いる.
したがって z2 , z3 を通る直線に関して w1 , w2 が対称の位置にあることを示せばよい.
一般に原点と α を結ぶ直線に関して z と対称な点は
( )
z
α
α
である.そこで
(
)
w1 − z2
(z3 − z2 ) =
z3 − z2
(
)
w1 − z2
(z3 − z2 )
z3 − z2


 z1 + z3 
 (z − z2 )
= 
 1
1  3
−
z3
z2
= −z2 z3 (z1 + z3 ) = w2 − z2
したがって,z2 , z3 を通る直線に関して w1 と w2 が対称の位置にある.つまり 直線 z2 z3 上
に点 z1 から下ろした垂線との交点は,点 w1 と w2 を結ぶ線分の中点であることが示された.
[注意]これは, w1 と w2 の中点が直線 z2 z3 上にあることを示すことでもよい.
5.2.4
名大前期文系
(1) A(α, α2 ) , B(β, β 2 ) とおく.
t = αβ + α2 β 2 =
59
)2
(
1
1
−
αβ +
2
4
ここで αβ はすべての実数値をとりうる.だから,t のとりうる値の範囲は
t>
=−
1
4
(2) t = 2 より
αβ + α2 β 2 = 2
これから (αβ − 1)(αβ + 2) = 0 なので αβ = 1, −2 .
一方 P(X, Y ) とおくと
X = α + β,
Y = α2 + β 2
これから Y = (α + β)2 − 2αβ = X 2 − 2αβ
(i) αβ = 1 のとき.Y = X 2 − 2 .さらに α, β は
t2 − Xt + 1 = 0
の解なので,これが実数であるために
X2 − 4 >
=0
これらが (X, Y ) のみたすべき必要十分条件である.ゆえに求める軌跡は
y = x2 − 2 (x <
= −2, 2 <
= x)
(ii) αβ = −2 のとき.Y = X 2 + 4 .さらに α, β は
t2 − Xt − 2 = 0
の解であるが,これはつねに実数解である.
ゆえに求める軌跡は
y = x2 + 4
5.2.5
一橋
(1) 条件 a|x| + |y| <
= a を満たす点の集合のなす領域を D とする.領域 D は x 軸, y 軸に関
して対称である.x >
= a となる.ゆえに領域は点 P(1, 0) ,点
= 0 のときは ax + y <
= 0, y >
Q(0, a) ,点 R(−1, 0) ,点 S(0, −a) を頂点とするひし形の周と内部からなる領域である.
y − (x + 1)2 = k とおく.k のとりうる範囲は放物線 y = (x + 1)2 + k が領域 D と共有点を
もつ範囲として定まる.
放物線は軸が負で k が変化すれば y 軸に平行に移動するので,k が最小になるのは放物線が
点 P(1, 0) か点 S(0, −a) を通るときである.
点 P(1, 0) を通るとき. k = −4 .点 S(0, −a) を通るとき. k = −a − 1 .
−4 > −a − 1 ⇐⇒ 3a なので,求める最小値は
{
−4
(0 < a <
= 3 のとき)
−a − 1 (3 < a のとき)
60
となる.
y
O
y
x
O
61
x
(2) 同様に考える.
y = (x + 1)2 + k が直線 QR ,つまり y = ax + a と接するときの k の値と接点を求める.
(x + 1)2 + k = ax + a より x2 − (a − 2)x + k + 1 − a = 0
ゆえに D = (a − 2)2 − 4k − 4 + 4a = 0 より k =
a−2
> −1 .
2
接点が線分 QR にあるとき.つまり −1 <
a2
.このとき接点は軸の位置で x =
4
a−2
< 0 のとき.
2 =
a2
.
4
a−2
接点が線分 QR にないとき.つまり 0 <
のとき.
2
a の範囲は 0 < a <
= 2 でこのとき最大値 k =
a の範囲は 2 < a で最大になるのは,点 Q を通るとき.このとき k = a − 1 .
ゆえに求める最大値は
 2
 a
(0 < a <
= 2 のとき)
4

a − 1 (2 < a のとき)
となる.
y
O
x
O
x
広大後期理
5.2.6
(1)
y
̸
→
−
−
AOB = 60◦ ,|→
a | = 1, | b | = 1 なので,O を原点にとり,A(1, 0, 0),B(cos 60◦ , sin 60◦ , 0)
−
→ →
−
として一般性を失わない.そして C(s, t, u) とする.この 3 座標が,3 ベクトル →
a, b, −
c
の成分である.
62
−
→
−c = 0 より s = 0.|→
−c | = 1 より t2 + u2 = 1.よって t = cos α,u = sin α とおける.た
a ·→
だし点 C は xy 平面にはないので 0 < α < 180◦ である.このとき
√
3
◦
cos α
p = cos α · sin 60 =
2
−1 < cos α < 1 なので,
√
√
3
3
−
<p<
2
2
[別解] ̸ BOC = γ (0 < γ < 180◦ ) とおく.̸ AOB = 60◦ ,̸ AOC = 90◦ なので,これら 3 つ
の角が空間の頂点を成立させる必要十分条件から
60◦ + 90◦ > γ
90◦ + γ > 60◦
γ + 60◦ > 90◦
60◦ + 90◦ + γ < 360◦
これから 30◦ < γ < 150◦ .
−
→ →
p = | b ||−
c | cos γ = cos γ なので
√
√
3
3
−
<p<
2
2
−
→
−
→
−→
→ −→ −→
−
−c とおく.−
→
→
(2) BH = s→
a + t b + u→
OH = OB + BH = s−
a + (1 + t) b + u−
c .H は △OAB で定
まる平面上にあるので 1 + t = 0.
−→ −→
−→ −→
また BH · OA = 0, BH · OC = 0 より,
1
−
→
→
→
→
(s−
a + t b + u−
c)·−
a =s+ t=0
2
−
→
→
→
→
(s−
a + t b + u−
c)·−
c = pt + u = 0
これから s =
1
, t = −1, u = p.
2
∴
−
→
−→ 1 −
→
BH = →
a − b + p−
c
2
(3) 四面体 OABC の体積を S とする.△OAC の面積は
1√ −
→
→
−c )2 = 1
|→
a |2 |−
c |2 − ( −
a ·→
2
2
である.したがって
S
=
=
=
=
1 1 −→
· · |BH|
3 2
√
2
1 1 −
−
→
→
→
−
a − b + p c 6
2
√
1 1
→
−
−
→ −
→
→
→
+ 1 + p2 − 2 · 2(−
a · b ) − 2p( b · →
c ) + 2 · 2p(−
c ·−
a)
6 4
√
√
1 1
1
1
3
+ 1 + p2 − − 2p2 =
− p2
6 4
2
6 4
63
√
3
これが最大になるのは p = 0 のときで最大値は
である.
12
5.2.7
福井医大
(1) 放物線 y =
x2
x
上の点 (2t, t2 ) における接線の傾きは y ′ = より t である.t ̸= 0 のとき法
4
2
線は
1
y = − (x − 2t) + t2
t
つまり
t3 + (2 − y)t − x = 0
···⃝
1
である.t = 0 のとき法線は x = 0 であるが ⃝
1 は,この場合を含んでいる.
⃝
1 が点 (p, q) を通るなら
t3 + (2 − q)t − p = 0
···⃝
2
となる.
3 点 A, B, C における法線が 1 点 (p, q) で交わるとする.このとき,a, b, c は ⃝
2 の異な
る 3 つの解である.したがって解と係数の関係から
a+b+c=0
である.
逆に a + b + c = 0 であるとき
ab + bc + ca = 2 − q, abc = p
···⃝
3
であるように p と q を定める.この p と q に関して,a, b, c は ⃝
2 の異なる 3 つの解である.
これは A, B, C における法線がすべて (p, q) を通ることを意味している.
よって 3 点 A, B, C における法線が 1 点で交わるための必要十分条件が a + b + c = 0 であ
ることが証明された.
−→ −→
(2) P(p, q) とする.̸ ACB が直角とすると CA · CB = 0 である.つまり
(2a − 2c)(2b − 2c) + (a2 − c2 )(b2 − c2 ) = 0
ところが,a − c ̸= 0, b − c ̸= 0 なので
4 + (a + c)(b + c) = 0
a + c = −b, b + c = −a なので,4 + ab = 0 となる.
4 + ab = 0, 4 + bc = 0, 4 + ca = 0
のいずれかが成り立てばよいので
(4 + ab)(4 + bc)(4 + ca) = 0
64
···⃝
4
⃝
4 より
64 + 16(ab + bc + ca) + 4(ab2 c + bc2 a + ca2 b) + a2 b2 c2 = 0
ところが ab2 c + bc2 a + ca2 b = abc(a + b + c) = 0 なので
64 + 16(ab + bc + ca) + a2 b2 c2 = 0
ここに ⃝
3 を代入する.64 + 16(2 − q) + p2 = 0 より
q=
1 2
p +6
16
···⃝
5
一方,t の 3 次方程式 ⃝
2 が異なる 3 つの実数解を持たなければならない.⃝
2 の左辺を f (t)
とおく.f ′ (t) = 3t2 + 2 − q .ゆえに 2 − q < 0 が必要で,さらに
( √
) (√
)
q−2
q−2
f −
f
<0
3
3
である.これから
p2 <
4
(q − 2)3
27
ゆえに求める軌跡は ⃝
5 , ⃝
6 より
y=
(
1 2
x + 6,
16
x2 <
···⃝
6
4
(y − 2)3
27
)
数列
6
6.1
6.1.1
問題
東大理系後期
区間 [0, 1] において関数 f (x) を
f (x) =



 2x


 −2x + 2
(
)
1
x<
=2
(
)
1
x>
2
とおく. 0 <
= a1 <
= 1 をみたす実数 a1 を初期値として数列 {an } を
an = f (an−1 ) (n = 2, 3, · · ·)
で定める.このとき次の問いに答えよ.
(1) f (b) = b を満たす, 0 <
= 1 なる実数 b をすべて求めよ.
=b<
(2) a4 が (1) で求めた b の値の 1 つに等しくなるような初期値 a1 をすべて求めよ.
(3) 条件「ある n >
= 1 に対して,an が (1) で求めた b の値の 1 つに等しくなる」をみたす初期
値は a1 はどのような実数としてあらわされるか.
3
(4) 初期値は a1 が (3) の条件をみたさないとき,an >
= 4 となるような n >
= 1 が存在すること
を示せ.
(5) 数列 {an } が收束するために初期値は a1 がみたすべき必要十分条件を求めよ.
65
6.1.2
お茶大前期
数列 {xn } が次の漸化式を満たしている.xi+1 =
x2i + 1
(i = 1, 2, 3, · · ·)
2
(1) すべての自然数 i に対して, xi+1 >
= xi が成り立つことを示せ.
(2) |x1 | <
= 1 のとき,すべての自然数 i に対して xi <
= 1 であることを示せ.
1∑
(xi − 1)2 が成り立つことを示せ.
2 i=1
n
(3) 自然数 n に対して,等式 xn+1 − x1 =
n
2
(4) |x1 | <
= 1 のとき,xn+1 − x1 >
= 2 (xn − 1) が成り立つことを示せ.
(5) 初項 x1 の値に応じて,数列 {xn } の収束,発散について調べ,収束するときは極限値を求
めよ.
6.2
6.2.1
解答
東大理系後期
1
1
2
(1) b <
= 2 かつ 2b = b または, 2 < b かつ −2b + 2 = b より b = 0, 3 .
(2) 一般に 0 <
= b < 1 のとき, f (x) = b となる x は,
b
b
, 1−
2
2
で, b = 1 なら f (x) = 1 となる x は x =
1
である.
2
これにもとづいて an+1 = f (an ) より a4 からはじめて順次 n の小さいものを作っていく.
a4
=
0,
a3
=
0,
a2
=
0,
a1
=
0,
1,
∴
2
3
1 2
, , 1
3 3
1
2
3
4
5
,
,
,
,
, 1
3·2 3·2 3·2 3·2 3·2
2
3
4
5
6
1
,
,
,
,
,
2
2
2
2
2
3 · 2 3 · 2 3 · 2 3 · 2 3 · 2 3 · 22
11
10
9
8
7
,
,
,
,
2
2
2
2
3 · 2 3 · 2 3 · 2 3 · 2 3 · 22
a1 =
j
12
(j = 0, 1, · · · , 12)
(3) 同様に
a1 =
j
3 · 2n−2
(j = 0, 1, · · · , 3 · 2n−2 )
と推測される.これを数学的帰納法で示す.
66
n = 1 のときは (1) より成立している.
n = k のとき成立しているとする. n = k + 1 のときの a2 がとりうる値は n = k のときの
a1 がとりうる値に等しい.ゆえに n = k のときの仮定より
a2 =
j
3 · 2k−2
(j = 0, 1, · · · , 3 · 2k−2 )
である.したがって
a2
=
j
(j = 0, 1, · · · , 3 · 2k−2 )
3 · 2k−1
3 · 2k−1 − j
(j = 0, 1, · · · , 3 · 2k−2 − 1)
3 · 2k−1
つまり
a1 =
j
3 · 2k−1
(j = 0, 1, · · · , 3 · 2k−1 )
となり,n = k + 1 でも成立する.
∴
a1 =
j
3 · 2n−2
(j = 0, 1, · · · , 3 · 2n−2 )
(4) 背理法で示す.すべての n に対して an <
3
とする.
4
このときある番号 n で 初期値 a1 は (3) のいずれかに一致するものが存在すること示せばよ
2
い.つまりある番号 n0 で an0 = 0 または an0 = となるものが存在すること示せばよい.
3
1
1
a1 = 0 なら存在した.もし 0 < a1 <
= 2 なら a2 = 2a1 と, 2 以下であるかぎり順次 2 倍さ
1
3
れていくので,必ずある番号 n0 で < an0 < となるものがある.さらにこのとき
2
4
an0 +1 = −2an0 + 2 <
3
4
なので
3
5
< an0 <
8
4
···⃝
1
である.したがって同様に
3
5
< an0 +1 <
8
4
···⃝
2
である.⃝
1 なら ⃝
2 がつねに成り立つので n >
= n0 に対して
5
3
< an <
8
4
である.ゆえにこの範囲の n に対しては漸化式 an+1 = −2an + 2 が成立する.
これから
an+1 −
(
)
2
2
= −2 an −
3
3
67
つまり
an −
ここで an <
2
= (−2)n−n0
3
3
なので
4
(−2)n−n0
an0 −
2
3
)
(
)
2
2
3
an 0 −
+ <
3
3
4
これから
(
n−n0
(−2)
(
an0
2
−
3
)
<
1
12
これが n >
= n0 であるすべての n で成立しなければならない.
2
もし an0 − ̸= 0 なら
3
(
)
2 n−n0
lim (−2)
an0 −
=∞
n→∞ 3 なので,十分大きい n で n − n0 が偶数になるものについては
(
)
2
1
n−n0
(−2)
an0 −
<
3
12
が成立しない.
2
ゆえに an0 = .つまり番号 n0 に対し an0 が (1) の値に一致した.ゆえに a = 1 は (3) の
3
条件をみたすことになり,背理法が成立した.
(5) 数列 {an } の初期値 a1 が (3) の条件をみたせば,ある番号 n0 があって n >
= n0 ならつねに
2
an = 0 か,つねに an = となる.つまり数列 {an } は收束する.
3
数列 {an } の初期値 a1 が (3) の条件をみたさないとする.
3
(4) より,ある番号 n1 で an1 >
= 4 となるものがある.
1
このとき an1 +1 = −2an1 + 2 より an1 +1 <
= 2 となる.
3
ここでもし n > n1 なる n に対してつねに an < なら,(4) と同様の議論によって,ある
4
2
n0 で an0 = 0,
となるものがあり,数列 {an } の初期値 a1 が (3) の条件をみたさなければ
3
ならない.
3
1
ゆえに n2 > n1 で an2 >
= 4 となるものがあり,an2 <
= 2 となる.
3
これをくり返せば, n1 < n2 < n3 < · · · で an1 , an2 , an3 , · · · >
= 4 となるものが存在し,こ
1
のとき an1 +1 , an2 +1 , an3 +1 , · · · <
= 2 である.
3
3
つまり数列 {an } の項で an >
= 4 となるものと, an <
= 4 となるものが無数に存在する.
数列 {an } は発散部分列を含むので,発散する.
よって数列 {an } が收束するための必要十分条件は数列 {an } の初期値 a1 が (3) の条件を満
たすことである.
68
6.2.2
お茶大前期
(1)
xi+1 − xi =
x2i + 1
(xi − 1)2
>
− xi =
=0
2
2
ゆえにすべての自然数 i に対して, xi+1 >
= xi が成り立つ.
(2) 数学的帰納法で |xi | <
= 1 を示す.
n = 1 なら成立.
n = i で |xi | <
= 1 とする.ゆえに
1
1
xi 2 + 1
<
<
1 .つまり <
|xi+1 | <
=
=
= 1 なので n = i + 1
2
2
2=
でも成立した.
(3) xi+1 − xi =
(xi − 1)2
より
2
n
∑
(xi − 1)2
i=1
2
=
n
∑
(xi+1 − xi ) = xn+1 − x1
i=1
ゆえに,自然数 n に対して,等式
1∑
(xi − 1)2
2 i=1
n
xn+1 − x1 =
が成り立つ.
(4) 関数 (x − 1)2 は x <
= 1 で単調減少である.(1) より xn >
= xi (i = 1, 2, 3, · · · , n) なので
2 >
2
(xi − 1) = (xn − 1) (i = 1, 2, 3, · · · , n) が成り立つ.
n
2
ゆえに (2) から |x1 | <
= 1 のとき,xn+1 − x1 >
= 2 (xn − 1) が成り立つ.
69
(5) |x1 | <
= 1 のとき.常に |xi | <
= 1 なので
−2 <
= xn+1 − x1 <
=2
(4) から
lim
n→∞
2(xn+1 − x1 )
2
>
= (xn − 1) >
=0
n
2(xn+1 − x1 )
= 0 から
n
lim (xn − 1)2 = 0
n→∞
つまり数列 {xn } は收束し, lim xn = 1 である.
n→∞
|x1 | > 1 のとき.(2) の証明と同様に |xi | > 1 (i = 1, 2, 3, · · ·) が成り立つ.
(3) の
1∑
(xi − 1)2
2 i=1
n
xn+1 − x1 =
はこの場合も成立する.
2
i>
= 2 に対しては xi > 0 なので 1 < xi である.x > 1 で関数 (x − 1) は単調増加である.
∴
xn+1 − x1 >
n−1
(x1 − 1)2
(x2 − 1)2 +
2
2
ゆえに数列 {xn } は発散する.
7
7.1
7.1.1
確率
問題
東大理系後期総合 II
(1) ランダムウオークと呼ばれる確率過程を考えよう.1 次元の数直線上をランダムに移動する
人を考える.時刻 0 において,数直線の原点に人がいるものとする.この人は時間ステップ
1
が 1 進むごとに+または−方向に 1 だけ移動するものとし,同じ確率,すなわち
の確率
2
1
1
で+方向に, の確率で−方向に動くものとする.次の時間ステップでは,再び, ずつの
2
2
確率で+または−方向を選んで移動し,これを繰り返すものとする.この過程において
(i) 時刻 0 から時間ステップが 3 進んだときに,この人が存在する可能性のある場所とそこ
にいる確率を求めよ.
(ii) 時刻 0 から時間ステップが 6 進んだときに,この人が原点にいる確率を求めよ.
(iii) 時刻 0 から時間ステップが 6 進んだときに,スタートしてから初めてこの人が原点に
戻ってくる確率を求めよ,
70
(2) ランダムウオークの応用として,ギャンブラ―の破産問題と呼ばれるものを考える.あるギャ
ンブラーの最初の所持金を 2 ドルとする.l 回賭けを行うごとに,勝てば所持金が 1 ドル増
え,負ければ 1 ドル失うものとする.所持金がなくなればギャンブラーは破産しそこで賭け
2
は終わり、また、所持金が 5 ドルになれば賭けは終了とする.1 回の賭けで勝つ確率を ,負
3
1
ける確率を としたとき,このギャンブラーが破産して終了する確率はいくらとなるかを考
3
えてみよう.この問題を数直線上で考えると,数直線上の位置 2 からスタートし,0 と 5 の
2
1
間で+方向ヘ移動する確率が ,−方向へ移動する確率が のらランダムウォークとなる.
3
3
0 または 5 に達したときに賭けは終了する.
さて,ランダムウオークでは,状態の変化は確率的に定まり,その確率は一定ある.すな
わち,ある状態にある場合、それ以降の過程は時刻や履歴にはよらず確率的に定まる,し
たがって,ギャンブラ―の破産確率は,単にそのときの所持金の額だけで決まる.これを
r(i) (i = 0, 1, · · · , ,5) としよう.r(i) は所持金が i であるときに,破産して終了する確率
である.
(i) r(2) を r(1), r(3) で表せ.
(ii) このような関係式を各状態について表し,それらを用いて最初の所持金が 2 ドルのギャ
ンブラーが破産して終了する確率を求めよ.
7.1.2
九大前期理系
平面上の点の x 座標と y 座標がどちらも整数であるとき,その点を格子点という.
与えられた格子点を第 1 番目とし,この点から右斜
y
め 45◦ ,または右斜め −45◦ の方向にもっとも近い第
2 番目の格子点をとり,この 2 点を線分で結ぶ.同
様にして第 2 番目の格子点から第 3 番目の格子点を
とり,第 2 番目と第 3 番目を線分で結ぶ.以下これ
を有限回繰り返し,こうしてできる線分をつないだ
ものを折れ線グラフということにする.右図に原点
O と格子点 (9, −1) を結ぶ折れ線グラフの例を示す.
次の問いに答えよ.
3
2
1
x
O
-1
-2
(1) n は正の整数, k は 0 <
=k<
= n なる整数とする.原点 O と格子点 (n, k) を結ぶ折れ線グラ
フが存在するための必要十分条件は n + k が偶数であることを示せ.またこの必要十分条件
がみたされているとき,原点 O と格子点 (n, k) を結ぶ折れ線グラフの数を求めよ.
(2) n は 2 以上の整数, k は 0 <
= n − 2 なる整数で,n + k は偶数とする.原点 O と格子点
=k<
(n, k) を結ぶ折れ線グラフであって格子点 (0, k), (1, k), · · · , (n − 2, k) の少なくとも 1 つ
を通る折れ線グラフの数は,原点 O と格子点 (n − 1, k + 1) を結ぶ折れ線グラフの数の 2 倍
に等しいことを示せ.
(3) コインを 9 回投げる.1 回から i 回までの試行において,表の出た回数から裏の出た回数
を引いた数を Ti で表す.このとき各格子点 (i, Ti ), i = 0, 1, 2, · · · , 9 を順番に線分で
つなげば折れ線グラフが得られる.ただし,T0 = 0 とする。T9 = 3 が起きたとき,どの
Ti (i = 1, 2, · · · , 7) も 3 にならない条件つき確率を求めよ.
71
7.1.3
大阪女子大前期理系
3 つの箱 A0 , A1 , A2 と 1 つの球があり,球は 3 つの箱 A0 , A1 , A2 の 1 つに入っている.球を
1
そのときに入っている箱から他の 2 つの箱のどちらかに確率
で移動させる.最初に球は箱 A0
2
に入っているものとし, k 回移動させた後に球が箱 An に入っている確率を Pn (k) とする.Pn (k)
を用いて Qn (k) を
)
(
2π
2π
n
n
+ i sin
Qn (k) = P0 (k) + ω
¯ P1 (k) + ω P2 (k) ω = cos
3
3
と定義する.ただし, ω
¯ は ω に共役な複素数である.このとき,次の問いに答えよ.
(1) P0 (1), P0 (2), P0 (3), P0 (4) を求めよ.
(2) n = 0, 1, 2 に対して,次の式が成立することを示せ.
Pn (k) =
1
{Q0 (k) + ω n Q1 (k) + ω
¯ n Q2 (k)}
3
(3) n = 0, 1, 2 に対して, Qn (k + 1) を Qn (k) を用いて表せ.
(4) P0 (k) を k を用いて表せ.
7.1.4
北大前期理系
(1) 1000 から 9999 までの 4 桁の自然数のうち,1000 や 1212 のようにちょうど 2 種類の数字か
ら成り立っているものの個数を求めよ.
(2) n 桁の自然数のうち,ちょうど 2 種類の数字から成り立っているものの個数を求めよ.
7.1.5
名大後期理系
赤と白のランプが一つずつある.それぞれのランプは,1 秒ごとにある規則にしたがって点灯ま
たは消灯し,1 秒間その状態を保持する.時刻 0 秒から実験を開始する.以下,m, n は 0 以上の
整数とする.
(1) 両方のランプとも,以下の規則 R1 , R2 にしたがうとする.
R1:時刻 m 秒からの 1 秒間に点灯している場合,時刻 m + 1 秒で消灯する確率は p である.
R2:時刻 m 秒からの 1 秒間に消灯している場合,時刻 m + 1 秒で点灯する確率は p である.
時刻 0 秒からの 1 秒間,白は消灯,赤は点灯しているとする.このとき,つぎの各問に答えよ.
(i) 時刻 n 秒からの 1 秒間に白のランプが点灯している確率を求めよ.
(ii) 時刻 n 秒からの 1 秒間に少なくとも一方のランプが点灯している確率を求めよ.
(2) つぎに,規則を以下の R1′ , R2′ , R3′ のように変更した.
R1′ :時刻 m 秒からの 1 秒間,両方が消灯している場合には,時刻 m + 1 秒ではどちらか一
方が点灯する.白が点灯する確率は p である.
R2′ :時刻 m 秒からの 1 秒間,どちらか一方だけが点灯している場合には,時刻 m + 1 秒で
残りのランプが点灯するか,あるいは,点灯しているランプが消灯する.残りのランプが点
72
灯する確率は p である.R3′ :時刻 m 秒からの 1 秒間,両方が点灯している場合には,時刻
m + 1 秒ではどちらか一方が消灯する.赤が消灯する確率は p である.
時刻 0 秒からの 1 秒間,白は消灯,赤は点灯しているとする.このとき,つぎの各問に答えよ.
(i) 時刻 n 秒からの 1 秒間に白のランプが点灯している確率を求めよ.
(ii) 時刻 n 秒からの 1 秒間に少なくとも一方のランプが点灯している確率を求めよ.
7.1.6
慶応医 2 番
四角形の 4 つの頂点に 1,2,3,4 と時計まわりに番号がつけられ
1
2
ている。時刻 0 において,この四角形の頂点 1 と頂点 3 の上をそれぞ
れ1つずつの粒子が占めているとし,頂点 2 と頂点 4 の上には粒子
図1
は存在しないものとする (図 1 を参照のこと).その後,1 秒ごとに,
1
存在する粒子の中で最小の番号の頂点上を占める粒子が,確率 で
2
4
3
1
消滅し,確率 ずつで隣り合う 2 つの頂点のいずれかに移動する.ただし,移動した頂点をすで
4
に他の粒子が占めている場合は,その粒子と合体して 1 つの粒子になるものとする.以下,m, n
を自然数とする.時刻 n(秒) において,この四角形の 4 つの頂点のうち 1 つの頂点上にのみ粒子が
存在する確率を Pn で表し,4 つの頂点のいずれの上にも粒子が存在しない確率を Qn で表す.
(1) P2 ,Q2 を求めよ.
(2) P2m−1 ,P2m ,Q2m−1 ,Q2m を求めよ.
7.2
7.2.1
(1)
解答
東大理系後期総合 II
(i) 時刻 0 から時間ステップが 3 進んだときに,この人が存在する可能性のある場所は
−3, −1, 1, 3 で一般に +1 が k 回,−1 が 3 − k 回となる確率 pk は
( )k ( )3−k
1
1
pk = 3 C3
2
2
である.それぞれ k = 0, 1, 2, 3 なのでその確率は順に
1 3 3 1
, , ,
8 8 8 8
(ii) 同様に
( )3 ( )3
1
1
5
=
6 C3
2
2
16
(iii) はじめに正の方に動いたときは,時刻 0 から時間ステップが 6 進んだときに,スター
トしてから初めてこの人が原点に戻ってくる事象は 0 → 1 → 2 → 3 → 2 → 1 → 0 と
0 → 1 → 2 → 1 → 2 → 1 → 0 のふた通り.はじめに負の方に動いたときも同様.この
4 通りのみ.したがって
1
4
=
26
16
73
(2) 破産の問題は漸化式
r(k) =
1
2
r(k + 1) + r(k − 1)
3
3
···⃝
1
を条件 r(0) = 1, r(5) = 1 のもとで解くことに他ならない.
(i)
r(2) =
2
1
r(3) + r(1)
3
3
(ii) ⃝
1 より
1
1
r(k + 1) − r(k) = r(k) − r(k − 1)
2
2
1
r(k + 1) − r(k) = (r(k) − r(k − 1))
2
これから
1
1
r(k + 1) − r(k) = r(1) − r(0)
2
2
( )k
1
r(k + 1) − r(k) =
(r(1) − r(0))
2
( )k
1
1
1
∴
r(k) = r(1) − −
(r(1) − 1)
2
2
2
r(5) = 0 なので
r(1) −
1
−
2
( )5
1
(r(1) − 1) = 0
2
15
31
{
( )2 (
)}
15 1
1
15
7
r(2) = 2
− −
−1
=
31 2
2
31
31
∴
∴
[注意]確率
7.2.2
r(1) =
2
に替えて一般に p としてこの問題を考えるとよい.
3
九大前期理系
(1) 原点と (n, k) を +45◦ が p 回, −45◦ が q 回で結ぶ折れ線グラフが存在することは,
p + q = n, p − q = k
となる正整数 p, q が存在することと同値である.
2p = n + k, 2q = n − k
n + k と n − k がともに偶数であることと同値になるが n − k = n + k − 2k より n + k と
n − k の偶数奇数は一致する.
ゆえに n + k が偶数であることが,折れ線グラフが存在するための必要十分条件である.
求める折れ線グラフの数は
p+q Cp
74
= n C n+k
2
(2) 原点と (n, k) を結ぶ折れ線で,点 (n, k) と結ばれる点は (n − 1, k + 1) か (n − 1, k − 1) の
いずれかである.
原点 O から (n − 1, k + 1) を通り (n, k) に至るグラフを一つとる.これは必ず格子点
(0, k), (1, k), · · · , (n − 2, k) の少なくとも 1 つを通る.
このグラフが格子点 (0, k), (1, k), · · · , (n − 2, k) のうち最後に通った点を (j, k) とする.
この折れ線グラフの点 (j, k) から (n, k) の部分を直線 y = k に関して折り返すと原点 O と
格子点 (n, k) を結ぶ折れ線グラフであって格子点 (0, k), (1, k), · · · , (n − 2, k) の少なく
とも 1 つを通りさらに (n − 1, k − 1) から (n, k) に至るグラフが一つできる.
この対応は一対一である.
原点 O と格子点 (n, k) を結ぶ折れ線グラフであって格子点 (0, k), (1, k), · · · , (n − 2, k)
の少なくとも 1 つを通るものが,(n − 1, k + 1) を通るものと (n − 1, k − 1) を通るものに
二分され同数ある.
原点 O から (n − 1, k + 1) を通るグラフはすべて格子点 (0, k), (1, k), · · · , (n − 2, k) の
少なくとも 1 つを通る.
ゆえに原点 O と格子点 (n, k) を結ぶ折れ線グラフであって格子点 (0, k), (1, k), · · · , (n−2, k)
の少なくとも 1 つ通る折れ線グラフの数は,原点 O と格子点 (n − 1, k + 1) を結ぶ折れ線グ
ラフの数の 2 倍に等しい.
(3) 原点と点 (9, T9 ) = (9, 3) を結ぶ折れ線グラフで,少なくとも 1 回 (i, Ti ) (i = 1, 2, · · · , 7)
を通るものは,(2) から原点と (8, 4) を結ぶ折れ線グラフの 2 倍ある.つまり
28 C 8+4 = 28 C6
2
ゆえに求める確率は
1−
7.2.3
28 C6
2
1
=1− =
C
3
3
9 6
大阪女子大前期理系
(1) 題意から
P0 (k) + P1 (k) + P2 (k) = 1
である.ゆえに
1
P1 (k − 1) +
2
1
P1 (k) = P2 (k − 1) +
2
1
P2 (k) = P0 (k − 1) +
2
P0 (k) =
1
P2 (k − 1) =
2
1
P0 (k − 1) =
2
1
P1 (k − 1) =
2
1
{1 − P0 (k − 1)}
2
1
{1 − P1 (k − 1)}
2
1
{1 − P2 (k − 1)}
2
また
P0 (0) = 1, P1 (0) = P2 (0) = 0
75
···⃝
1
ゆえに
P0 (1) =
P0 (2) =
P0 (3) =
P0 (4) =
1
{1 − P0 (0)} = 0
2
1
1
{1 − P0 (1)} =
2
2
1
1
{1 − P0 (2)} =
2
4
3
1
{1 − P0 (3)} =
2
8
(2) ω は ω 3 = 1,ω 2 + ω + 1 = 0,ω
¯ = ω 2 が成り立つ.ゆえに n = 0 なら 1 + ω n + ω
¯n = 3 ,
n = 1, 2 なら 1 + ω n + ω
¯ n = 0 である.よって
=
1
{Q0 (k) + ω n Q1 (k) + ω
¯ n Q2 (k)}
3
1
{P0 (k) + P1 (k) + P2 (k)
3
+ω n P0 (k) + ω n+2 P1 (k) + ω n+1 P2 (k) + ω 2n P0 (k) + ω 2n+1 P1 (k) + ω 2n+2 P2 (k)}
=
1
{(1 + ω n + ω 2n )P0 (k) + (1 + ω n+2 + ω 2n+1 )P1 (k)
3
+(1 + ω n+1 + ω 2n+2 )P2 (k)}
···⃝
2
したがって
⃝
2 = P0 (k) (n = 0), ⃝
2 = P1 (k) (n = 1), ⃝
2 = P2 (k) (n = 2)
となり題意が示された.
(3)
Qn (k + 1)
=
=
=
P0 (k + 1) + ω
¯ n P1 (k + 1) + ω n P2 (k + 1)
1
ω 2n
ωn
{P1 (k) + P2 (k)} +
{P2 (k) + P0 (k)} +
{P0 (k) + P1 (k)}
2
2
2
ω 2n + ω n
1 + ω 2n
1 + ωn
P0 (k) +
P1 (k) +
P2 (k)
2
2
2
したがって
Q0 (k + 1) = P0 (k) + P1 (k) + P2 (k) = Q0 (k)
1
1
Q1 (k + 1) = {−P0 (k) − ωP1 (k) − ω 2 P2 (k)} = − Q1 (k)
2
2
1
1
2
Q2 (k + 1) = {−P0 (k) − ω P1 (k) − ωP2 (k)} = − Q2 (k)
2
2
(4) Q0 (0) = Q1 (0) = Q2 (0) = 1 なので (3) から
Q0 (k) = Q0 (0) = P0 (0) + P1 (0) + P2 (0) = 1
(
)k
(
)k
1
1
Q1 (k) = −
Q1 (0) = −
2
2
(
)k
(
)k
1
1
Q2 (k) = −
Q2 (0) = −
2
2
76
ゆえに
P0 (k)
=
=
[別解] ⃝
1 より,
1
{Q0 (k) + Q1 (k) + Q2 (k)}
3
{
{
(
)k }
(
)k−1 }
1
1
1
1
1+2 −
=
1− −
3
2
3
2
=
{
}
1
1
1
=−
P0 (k − 1) −
3
2
3
{
(
(
)k {
}
)k−1 }
1
1
1
1
1
P0 (k) = + −
1− −
P0 (0) −
=
3
2
3
3
2
P0 (k) −
∴
7.2.4
北大前期理系
(1) 2 種類の数字の一つが 0 である場合.2 種の数字の選び方は 9 通り,
0 でない数字を△とする.千の桁は△のみ.他の三桁のうち
3 桁とも 0 である場合 · · · 1 (個)
2 桁が 0 である場合
· · · 3 ( 個)
1 桁が 0 である場合
· · · 3 ( 個)
2 種とも 0 でない場合.数字の選び方は 9 C2 = 36 (通り).
2 つのうち一方が何回現れるかで分類する.並べ方は
4 C3
+ 4 C2 + 4 C1 = 4 + 6 + 4 = 14
9 × 7 + 36 × 14 = 727 (個)
∴
(2) 同様に考える.文字の選び方は (1) と同じである,並べ方は
1 つが 0 である場合.
n−1 Cn−2
+ · · · + n−1 C1 + n−1 C0 = 2n−1 − 1
0 がない場合.
n Cn−1
∴
7.2.5
+ n Cn−2 + · · · + n C1 = 2n − 2
9 × (2n−1 − 1) + 36 × (2n − 2) = 81(2n−1 − 1) (個)
名大後期理系
(1) n 秒からの 1 秒間に白が点灯している確率を wn ,赤が点灯している確率を rn とおく.w0 =
0, r0 = 1 である.
77
(i) 点灯から点灯,消灯から消灯の確率はともに,1 − p である.
wn+1
1 − wn+1
= p(1 − wn ) + (1 − p)wn
=
(1 − p)(1 − wn ) + pwn
wn+1 = p + (1 − 2p)wn
(
)
1
1
これから wn+1 − = (1 − 2p) wn −
.
2
2
(
)
1
1
1
n
∴ wn − = (1 − 2p)
w0 −
= − (1 − 2p)n
2
2
2
∴
∴
(ii) 同様に
rn −
wn =
1
{1 − (1 − 2p)n }
2
(
)
1
1
1
= (1 − 2p)n r0 −
= (1 − 2p)n
2
2
2
∴
rn =
1
{1 + (1 − 2p)n }
2
少なくとも一方のランプが点灯している事象は,両方消灯している事象の余事象である.
∴
=
1 − (1 − wn )(1 − rn ) = wn + rn − wn rn
} 3 1
1{
1 − (1 − 2p)2n = + (1 − 2p)2n
1−
4
4 4
(2) wn , rn は (1) と同様とし,さらに,n 秒からの 1 秒間に,両方点灯している確率を an ,一方
だけ点灯している確率を bn ,両方消灯している確率を cn とする.
(i) 条件から両方点灯しているとき白が点灯する確率は p.一方だけ点灯しているとき,も
し白が点灯していれば確率 p で白は点灯し続け,もし赤が点灯していればやはり確率 p
で白が点灯する.
∴
wn+1 = pan + pbn + pcn = p(an + bn + cn ) = p
{
0 (n = 0)
∴ wn =
p (n >
= 1)
(ii) 0 秒では一方だけが点灯している.以下次の過程をくりかえす.
両方が点灯
p
一方のみ点灯
一方のみ点灯
1−p
両方が消灯
したがって求める確率は
{
n が偶数なら 1
n が奇数なら p
である.
78
7.2.6
慶応医 2 番
(1) n 秒後に,この四角形の 4 つの頂点のうち,隣りあわない 2 つの頂点上に粒子が存在する確
率を Rn とし,隣りあう 2 つの頂点上に粒子が存在する確率を Sn とする.
n + 1 秒後に 1 つの頂点上にのみ粒子が存在するのは,n 秒後に,1 つの頂点上にのみ粒子が
存在してそれが移動するか,隣りあわない 2 つの頂点上に粒子が存在し 1 つが消滅するか,
隣りあう 2 つの頂点上に粒子が存在し 1 つが消滅するか 1 つが他と合体するか,そのいずれ
かである.他も同様に考え,次の漸化式が成り立つ.

1
3
1

 Pn+1 = Pn + Rn + Sn


2
2
4



1


 Qn+1 = Pn + Qn
2
1


Rn+1 = Sn



4



1

 Sn+1 = Rn
2
R0 = 1, P0 = Q0 = S0 = 0 より P1 =
P2 =
1
1
, Q1 = R1 = 0, S1 = なので
2
2
1 3
5
+ = ,
4 8
8
Q2 =
1
4
である.
(2) 漸化式から
Rn+2 =
となる.よって,
1
1
Sn+1 = Rn
4
8
( )m
( )m
1
1
R0 =
8
8
( )m−1
1
1
R2m+1 = R2m−1 より R2m−1 =
R1 = 0
8
8
( )m
1
1
1 1
S2m = R2m−1 = 0, S2m+1 = R2m =
2
2
2 8
R2m+2 =
この結果,
P2m+1
=
Q2m+1
=
P2m
=
Q2m
=
( )m
1
1
1
1 1
P2m + R2m = P2m +
2
2
2
2 8
1
P2m + Q2m
2
( )m
1
3
1
1
P2m−1 + S2m−1 = P2m−1 + 3
2
4
2
8
1
P2m−1 + Q2m−1
2
したがって,
P2m+1
1
R2m より R2m =
8
=
=
{
( )m }
( )m
1 1
1
1 1
P2m−1 + 3
+
2 2
8
2 8
( )m
1
1
P2m−1 + 2
4
8
79
つまり
4m P2m+1 = 4m−1 P2m−1 + 2
( )m
1
2
この階差数列を解く.
m−1
4
P2m−1
m−1
∑(
)k
1
= P1 + 2
2
k=1
{
( )m−1 }
( )m−1
1
5
1
1
+2 1−
= −2
=
2
2
2
2
よって
P2m−1 =
10
2
− m−1
4m
8
これより,
P2m
5
1
= m − m−1 + 3
4
8
( )m
1
5
5
= m− m
8
4
8
Pn + Qn + Rn + Sn = 1 なので
Pn+1 =
1
1
1
1
(Pn + Rn + Sn ) + Sn = (1 − Qn ) + Sn
2
4
2
4
つまり
1
Qn = 1 − 2Pn+1 + Sn
2
であるから
Q2m−1
Q2m
1
= 1 − 2P2m + S2m−1
2
( )m−1
10
10
1 1
10
12
= 1− m + m +
=1− m + m
4
8
4 8
4
8
1
= 1 − 2P2m+1 + S2m
2
20
4
5
4
= 1 − m+1 + m = 1 − m + m
4
8
4
8
となる.
8
8.1
8.1.1
曲線
問題
九大前期理系
{
x = f (θ) cos θ
(θ − π)2 1
+ を用いて
平面上の点 P の x 座標と y 座標が,変数 θ の関数 f (θ) =
2
2π
2
y = f (θ) sin θ
<
2π
の範囲で変化したとき,点
P
が描く曲線を
C
とする.
P を P(θ)
θ
と表されている.θ が 0 <
= =( )
π
で表し,P1 = P(0),P2 = P
,P3 = P(π) とおく.次の問いに答えよ.
2
80
(y − β)2
(x − α)2
+
= 1 (a > 0, b > 0) で与えられる楕円が点 P1 を通るとする.
a2
b2
このとき,点 P3 がこの楕円の内部に含まれる (ただし,楕円の上にない) ための必要十分条
件を α のみを用いて表せ.
(1) 方程式
(2) 点 P2 における曲線 C の接線を l とする. l の方程式を求めよ.
(3) 次の条件 (i),(ii),(iii) をみたす楕円 D を考える.
(i) D の軸の一つは x 軸上にある.
(ii) D は点 P1 , P2 を通る.
(iii) 点 P2 における D の接線は l である.
このとき,点 P3 は楕円 D の内部に含まれるかどうか判定せよ.
8.2
8.2.1
解答
九大前期理系
( ) (
)
(
)
π
5
1
(1) P1 = P(0) = (1, 0),P2 = P
= 0,
,P3 = P(π) = − , 0 なので条件は
2
8
2
)2
(
1
− −α
β2
2
+ 2 <1
2
a
b
(1 − α)2
β2
+ 2 = 1,
2
a
b
である.
これから
(
つまり
(
)2
1
− −α
(1 − α)2
2
<
a2
a2
1
+α
2
)2
< (1 − α)2 .これを解いて α <
1
.
4
(2)
d
x = f ′ (θ) cos θ − f (θ) sin θ,
dθ
また f ′ (θ) =
d
y = f ′ (θ) sin θ + f (θ) cos θ
dθ
θ−π
なので点 P2 における曲線 C の接線の傾きは
π2
( )
( )
π
π
π
π
f′
sin + f
cos
4
2
2
2
2
( )
( )
=
π
π
π
π
5π
cos − f
sin
f′
2
2
2
2
したがって l の方程式は
y=
4
5
x+
5π
8
81
(3) 条件 (i) から楕円 D は
(x − α)2
y2
+ 2 =1
2
a
b
とおける.P1 , P2 を通るので
(1 − α)2
02
+
= 1,
a2
b2
点 P2 での接線は
(0 − α)2
52
+
=1
a2
82 b2
···⃝
1
0−α
5
(x − α) + 2 y = 1
a2
8b
つまり
y=
8b2
8b2
αx +
2
5a
5
(
)
α2
1− 2
a
8b2
4
α=
5a2
5π
···⃝
2
4
5
x + と一致する.
5π
8
傾きを比較して
これが y =
⃝
1 ,⃝
2 より
α=
ここで
32
64 + 25π
32
1
< なので (1) より点 P3 は楕円 D の内部に含まれる.
64 + 25π
4
行列
9
9.1
問題
金沢大理系後期
(
)
(
1 0
1
√
,J=
行列 E =
0 1
2
9.1.1
√ )
√
− 2
1
3
に対して A = E +
J とおく.
3
3
−1
(
√ )n
1 + 3i
(1) sn , tn (n = 1, 2, 3, · · ·) は sn + itn =
を満たす実数とする.ただし, i は虚
3
数単位である.sn , tn を求めよ.また,これらの sn , tn に対して, An = sn E + tn J が成
り立つことを示せ.
(2) O を原点とする座標平面上の点列 Pn (xn , yn ) を
(
)
( )
xn
1
n
=A
(n = 1, 2, 3, · · ·)
yn
0
によって定め, △OPn Pn+1 の面積を Rn とする.
(i) Rn を求めよ.
∞
∑
(ii)
Rn を求めよ.
n=1
82
富山医科薬科
)
(
5
−1
2
とし,数列 {xn }, {yn } を
A=
1 0
(
) ( ) (
)
(
)
x0
1
xn
xn−1
=
,
=A
(n = 1, 2, · · ·)
y0
1
yn
yn−1
9.1.2
によって定義する.
(1) axn 2 + bxn yn + cyn 2 = −1 (n = 0, 1, 2) を満たすような定数 a, b, c を求めよ.
(2) (1) の定数 a, b, c は任意の n = 0, 1, 2, · · · に対して axn 2 + bxn yn + cyn 2 = −1 を満たす
ことを証明せよ.
3
(3) 任意の n = 0, 1, 2, · · · に対して 0 < yn <
= xn かつ xn+1 >
= 2 xn が成立することを証明せよ.
yn
を求めよ.
n→∞ xn
(4) lim xn , lim
n→∞
9.2
9.2.1
解答
金沢大理系後期
(1) 数列 {sn }, {tn } は
(
sn+1 + itn+1 =
より, 漸化式
sn+1
√ )n+1 (
√ )
1 + 3i
1 + 3i
=
(sn + itn )
3
3
√
√
sn − 3tn
3sn + tn
=
, tn+1 =
3
3
を満たす.これを用いて An = sn E + tn J が成り立つことを数学的帰納法で示す.
√
1
3
s1 = , t1 =
より n = 1 のときは成立.
3
3
(
√ )2
1 − 2
2
√
n で成立するとする.J =
= −E なので
2 −1
(
√ )
1
3
n+1
A
= (sn E + tn J)A = (sn E + tn J)
E+
J
3
3
√
√
sn − 3tn
3sn + tn
E+
J = sn+1 E + tn+1 J
=
3
3
ゆえに n + 1 でも成立し,一般に An = sn E + tn J が成り立つ.
(2)
(i)
(
xn
yn
)
(
n
= A
(
=
1
0
)
(
= (sn E + tn J)
sn + tn
√
2tn
83
)(
√
− 2tn
1
sn − tn
0
1
0
)
)
(
=
sn + tn
√
2tn
)
つまり Pn (xn , yn ) = (sn + tn ,
∴
Rn
√
2tn )
1
|xn yn+1 − xn+1 yn |
2
√
√
√
1
2
|(sn + tn ) 2tn+1 − 2tn (sn+1 + tn+1 )| =
|sn tn+1 − sn+1 tn |
2
2
√
√ √
sn − 3tn 2
3sn + tn
−
tn sn
2 3
3
√
√
6
6
(sn 2 + tn 2 ) =
|sn + itn |2
6
6
√ 2n
√ (
√ )n
6 1 + 3i 6 1 3
+
=
6 3
6
9 9
√ ( )n
6 4
6
9
= △OPn Pn+1 =
=
=
=
=
=
(ii)
√
√ ( )n
6 4
6 4
Rn =
=
· ·
6
9
6 9
n=1
n=1
∞
∑
9.2.2
∞
∑
√
2 6
=
4
15
1−
9
1
富山医科薬科
(1) n = 0, 1, 2 に対して (xn , yn ) は順に
(
(1, 1),
である.したがって条件は
) (
)
3
11 3
, 1 ,
,
2
4 2


a + b + c = −1



 9
3
a + b + c = −1
4
2


33
9
121



a + b + c = −1
16
8
4
これを解いて (a, b, c) = (2, −5, 2)
(2)

 x = 5x
n−1 − yn−1
n
2

yn = xn−1
···⃝
1
なので
axn 2 + bxn yn + cyn 2
(
)2
)
(
5
5
= a
xn−1 − yn−1 + b
xn−1 − yn−1 xn−1 + cxn−1 2
2
2
(
)
25
5
=
a + b + c xn−1 2 − (−5a − b)xn−1 yn−1 + ayn−1 2
4
2
84
(1) の a, b, c に対して
(
)
25
5
a + b + c xn−1 2 − (−5a − b)xn−1 yn−1 + ayn−1 2 = 2xn−1 2 − 5xn−1 yn−1 + 2yn−1 2
4
2
なので任意の n に対して
axn 2 + bxn yn + cyn 2 = axn−1 2 + bxn−1 yn−1 + cyn−1 2
が成立する.ゆえに任意の n = 0, 1, 2, · · · に対して axn 2 + bxn yn + cyn 2 = −1 が成立する.
3
(3) 0 < yn <
= xn かつ xn+1 >
= 2 xn が成立することを数学的帰納法で示す.
3
n = 0 のとき. y0 = x0 = 1, x1 = より成立.
2
3
k = k に対して 0 < yk <
= xk かつ xk+1 >
= 2 xk が成立するとする.
このとき
3
yk+1 = xk > 0, xk+1 >
= 2 xk > xk = yk+1 ∴
0 < yk+1 <
= xk+1
また
xk+2 =
3
5
5
xk+1 − yk+1 >
= 2 xk+1 − xk+1 = 2 xk
2
も成立する.
ゆえに任意の n = 0, 1, 2, · · · に対して題意が示された.
(4)
xn >
=
また xn ̸= 0 なので
(
2
0<
( )n
3
x0 ∴ lim xn = +∞
n→∞
2
yn
xn
yn
< 1 より
xn
1
yn
=
xn
4
∴
lim
n→∞
)2
−5
(
yn
1
+2+
=0
xn
xn
√
))
(
1
25 − 8 2 +
xn
5−
√
) 1
yn
1(
=
5 − 25 − 16 =
xn
4
2
[注] ⃝
1 を解いてしまっても以下の問に答えることができる.
85
微積
10
問題
10.1
10.1.1
(1)
千葉大理系後期
kθ + α
が整数とならないようなすべての実数 k, θ, α に対して
π
sin((k + 1)θ + α)
sin((k − 1)θ + α)
= 2 cos θ −
sin(kθ + α)
sin(kθ + α)
が成り立つことを示せ.
(
)
π
(2) p = cos θ 0 < θ <
とする.実数 α1 を α1 >
= p となるようにとり,
2
αj > 0 ならば,αj+1 = 2p −
1
αj
によって実数 α1 , α2 , · · · を順次定める.このとき,ある正の整数 n があって αj > 0 (j =
1, 2, · · · , n) かつ αn+1 <
= 0 になることを示せ.
(3) (2) において, θ =
10.1.2
π
とする. α1 の値がかわるとき,(2) の n の最大値を求めよ.
100
東工大前期
n を自然数とする.
(1) 次の極限を求めよ.
1
lim
n→∞ log n
(
)
1 1
1
1 + + + ··· +
2 3
n
(2) 関数 y = x(x − 1)(x − 2) · · · (x − n) の極値を与える x の最小値を xn とする.このとき
1
1
1
1
=
+
+ ··· +
xn
1 − xn
2 − xn
n − xn
1
および 0 < xn <
= 2 を示せ.
(3) (2) の xn に対して,極値 lim xn log n を求めよ.
n→∞
10.1.3
東北大前期理系
f1 (x) は実数全体で定義された何回でも微分可能な関数とする.
f2 (x), f3 (x), · · · を次のように
∫ x
順次定義する. n = 2, 3, · · · に対し, Fn−1 (x) =
fn−1 (t) dt とおいて
∫
fn (x) =
0
x
fn−1 (t)Fn−1 (t) dt
0
とする.このとき以下の問いに答えよ.
(1) n >
= 0 であることを示せ.
= 2 のとき,すべての x に対して fn (x) >
′
(2) n >
= 0 であることを示せ.
= 0 に対して fn (x) >
= 3 のとき,すべての x >
(3) f4 ′ (1) = 0 のとき,すべての 0 <
= 1 に対して f1 (x) = 0 であることを示せ.
=x<
86
10.1.4
名大前期理系
f (x) を実数全体で定義された連続関数で,
x > 0 で 0 < f (x) < 1 を満たすものとする.a1 = 1
∫ am−1
とし,順に,am =
f (x) dx (m = 2, 3, 4, · · ·) により数列 {am } を定める.
0
(1) m >
= 2 に対し, am > 0 であり,かつ a1 > a2 > · · · > am−1 > am > · · · となることを示せ.
(2)
1
> am となる m が存在することを背理法を用いて示せ.
2002
解答
10.2
10.2.1
千葉大理系後期
(1)
{
sin((k + 1)θ + α) = sin(kθ + α) cos θ + sin θ cos(kθ + α)
sin((k − 1)θ + α) = sin(kθ + α) cos θ − sin θ cos(kθ + α)
辺々加えて
sin((k + 1)θ + α) + sin((k − 1)θ + α) = 2 sin(kθ + α) cos θ
kθ + α
これから
が整数とならないようなすべての実数 k, θ, α に対して
π
sin((k + 1)θ + α)
sin((k − 1)θ + α)
= 2 cos θ −
sin(kθ + α)
sin(kθ + α)
が成り立つ.
(2) 関数 f (x) を f (x) =
sin(θ + x)
とする.
sin x
f (x) = sin θ
なので 0 < x <
1
+ cos θ
tan x
π
で f (x) は単調減少.
2
lim f (x) = +∞,
x→0
lim f (x) = cos θ
x→ π
2
π
なので,α1 >
= p となる α1 に対して f (x) = α1 となる x が 0 < x < 2 にただ一つ定まる.
この x の値を α とする.p > 0 より 0 < θ + α < π である.
αj > 0 ならば,αj+1 = 2p −
1
αj
のとき (1) から
αk =
sin(kθ + α)
sin((k − 1)θ + α)
となる.
0 < θ + α < π なので,ある正の整数 n があって jθ + α < π (j = 1, 2, · · · , n) かつ
π<
= (n + 1)θ + α となるものがある.
このとき sin(jθ + α) > 0 かつ sin((n + 1)θ + α) <
= 0 なので,
この n が αj > 0 (j = 1, 2, · · · , n) かつ αn+1 <
= 0 となるものである.
87
(3) (2) の n は nθ + α < π となる最大のものである.
α1 の値を変えるとそれに応じて α の値も変わる. α が小さいほど n は大きくとれる.
π
θ=
なので,α を十分0に近づければ n の最大値として 99 がとれる.
100
10.2.2
東工大前期
(1) 関数 y =
1
は単調減少なので自然数 k と区間 [k, k + 1] にある x に対して
x
1
1
1
> >
k
x
k+1
が成り立つ.これを k から k + 1 まで積分して
1
>
k
ゆえに
∫
k+1
k
1
1
dx >
x
k+1
∫ n+1
n
n ∫
∑
1 ∑ k+1 1
1
>
dx =
dx = log(n + 1)
k
x
x
1
k
1
k=1
また
∫ n
n
n−1
n−1
∑
∑ 1
∑ ∫ k+1 1
1
1
=1+
<1+
dx < 1 +
dx = 1 + log n
k
k+1
x
x
k
1
k=1
したがって
k=1
k=1
log(n + 1)
1 ∑1
1 + log n
1
<
<
=1+
log n
log n 1 k
log n
log n
n
···⃝
1
ここで
log n + log(1 +
log(n + 1)
=
log n
1
)
n
log n
log(1 +
=1+
log n
より n → ∞ のとき, ⃝
1 の左辺右辺は 1 に収束する.
1 ∑1
=1
n→∞ log n
k
n
∴
lim
k=1
(2) y = u1 u2 · · · un のとき
′
y =
n
∑
u1 · · · uk ′ · · · un
k=1
である.したがって
y′ =
y
y
y
+
+ ··· +
x x−1
x−n
···⃝
1
x = i のとき
y ′ (i) = (i − 1)(i − 2) · · · 1 · (−1) · · · (i − n) ̸= 0
なので xn ̸= 1, 2 · · · , n .
88
1
)
n
⃝
1 に x = xn を代入する.y = 0 となる x は x = 1, 2 · · · , n のみである.ゆえに x = xn
のとき y ′ = 0 かつ y ̸= 0 .これから
0=
1
1
1
+
+ ··· +
xn
xn − 1
xn − n
1
1
1
1
=
+
+ ··· +
xn
1 − xn
2 − xn
n − xn
∴
···⃝
2
y は x < 0 で単調であるから x < 0 に極値はない.また x = 0, 1 で 0 になる微分可能な関
数であるから Roll の定理より 0 < x < 1 に y ′ = 0 となる x があり,xn は最小のものなの
で 0 < xn < 1 である.
1
1
ここで xn > と仮定すると
< 2 である.一方
2
xn
1
1
1
1
+
+ ··· +
>
>2
1 − xn
2 − xn
n − xn
1 − xn
1
なので ⃝
2 と矛盾する.ゆえに 0 < xn <
= 2 が示された.
1
(3) 0 < xn <
= 2 より
1+
1
1
1
1
1 1
<
+ + ··· + <
+
+ ··· +
2 3
n
1 − xn
2 − xn
n − xn =
1
1
1−
2
+
1
1
2−
2
+ ··· +
ここで
1
1−
1
2
+
1
2−
1
2
1
+ ··· +
n−
1
2
=2+
2
2
+ ··· +
3
2n − 1
(1) と同様に
2+
∴
2
2
+ ··· +
<2+
3
2n − 1
∫
n
1
(
)
2
1
1
dx = 2 + log n −
− log
2x − 1
2
2
log n
log n
)
(
> xn log n >
=
1
1 1
1
1
1 + + + ··· +
− log
2 + log n −
2 3
n
2
2
(1) と同様に
lim
n→∞
log n
(
)
=1
1
1
2 + log n −
− log
2
2
なので
lim xn log n = 1
n→∞
89
1
n−
1
2
10.2.3
東北大前期理系
′
(1) n >
= 2 のとき Fn−1 (x) = fn−1 (x) および Fn−1 (0) = 0 である.ここで
∫ x
∫ x
′
fn−1 (t)Fn−1 (t) dt =
Fn−1
(t)Fn−1 (t) dt
fn (x) =
0
0
∫ x
1 d
=
· {Fn−1 (t)}2 dt
2
dt
0
[
]x
1
1
=
{Fn−1 (t)}2 = {Fn−1 (x)}2 >
=0
2
2
0
∫
(2) n >
= 3 のとき (1) から fn−1 >
= 0.また Fn−1 (x) =
x
0
fn−1 (t) dt より Fn−1 (x) >
= 0.
′
一方 fn ′ (x) = fn−1 (x)Fn−1 (x) であるから, n >
= 3 のときすべての x >
= 0 に対して fn (x) >
=0
である.
(3) f4 ′ (1) = 0 より
f3 (1)F3 (1) = 0
∫
1
ここで F3 (1) = 0 とする. F3 (1) =
0
f3 (t) dt で f3 (t) >
= 0 なので, F3 (1) = 0 から
f3 (x) = 0 (0 <
=x<
= 1) となる.したがっていずれにせよ f3 (1) = 0
1
{F2 (1)}2 であるから,F2 (1) = 0
2
∫ 1
F2 (1) =
f2 (t) dt で f2 (t) >
= 0 なので, F2 (1) = 0 から f2 (x) = 0 (0 <
=x<
= 1) となる.
f3 (1) =
0
1
f2 (x) = {F1 (x)}2 であるから, 0 <
=x<
= 1 で F1 (x) = 0
2
最後に f1 (x) = F1 ′ (x) より f1 (x) = 0 (0 <
=x<
= 1) である.
10.2.4
名大前期理系
(1) am > 0 であることを数学的帰納法で示す.
定義より a1 = 1 > 0 .am−1 > 0 とする. f (x) > 0 より
∫ am−1
f (x) dx > 0
am =
0
ゆえに m >
= 2 に対して f (x) < 1 なので
= 1 に対して am > 0 である.このとき m >
∫ am−1
∫ am−1
1 dx = am−1
f (x) dx <
am =
0
0
よって a1 > a2 > · · · > am−1 > am > · · · となることが示された.
(2)
1
1
< am が成り立つとす
> am となる m が存在しないと仮定する.つまりつねに
2002
2002 =
る.これから
1
< lim am · · · ⃝
1
2002 = m→∞
90
1
に一つとる.仮定から各 m に対して α < am である.f (x) は連
2002
続関数であるから α <
= x <
= 1 における最大値が存在する.それを M とする. x > 0 で
0 < f (x) < 1 なので 0 < M < 1 である.
∫ am−1
∫ am−1
<
f (x) dx =
M dx = M (am−1 − α)
次に α を 0 < α <
α
α
であるが
∫
∫
am−1
am−1
f (x) dx =
∫
α
∫
α
f (x) dx −
f (x) dx
∫
0
0
am−1
>
α
f (x) dx −
0
1 dx = am − α
0
なので
am − α < M (am−1 − α)
これから
am − α < M m−1 (a1 − α)
0 < M < 1 であるから
lim (am − α) = 0
m→∞
よって
lim am = α <
m→∞
⃝
2 は⃝
1 と矛盾する.したがって
1
2002
···⃝
2
1
> am となる m が存在することが示された.
2002
高校数学の方法関連問題
10.3
10.3.1
何をどう置くか
02 名工大
平面上に 三角形 ABC と 2 つの正三角形 ADB , ACE とがある.ただし,点 C ,点 D は直
線 AB に関して反対側にあり,また,点 B ,点 E は直線 AC に関して反対側にある.線分 AB の
中点を K,線分 AC の中点を L,線分 DE の中点を M とする.線分 KL の中点を N とするとき,
直線 MN と直線 BC とは垂直であることを示せ.
10.3.2
条件の同値変形
02 奈良女子大
a
b
a
(1) 0 以上の整数 a, b が a > b を満たすとき 2a−1 <
= 2 − 2 < 2 が成り立つことを示せ.
(2) 0 以上の整数 k, l, m, n が k > l, m > n かつ 2k − 2l = 2m − 2n を満たすとき, k = m か
つ l = n であることを示せ.
91
10.3.3
例で考える
02 滋賀医大
第1段
1
各頂点から 2 本ずつ枝分かれした樹形
3
2
第2段
図の各頂点に,上から下へ左から右へと
4
図のように番号をつけ番号 i をつけた
5
6
7
第3段
頂点を頂点 i と呼ぶ.
第4段
(1) 頂点 i のすぐ下の段にあって頂点 i と結ばれている頂点の番号は 2i と 2i + 1 であることを
示せ.
(2) 第 n 段までのすべての頂点に 1 つずつ石を置いて,各石を 1 段ずつ枝に沿って上に動かし,
頂点 1 に達すれば「上がり」として外に取り出す.全ての石を取り出すのに要する操作の総
数 Sn を求めよ.ただし,1 つの石を 1 段上に動かすこと,または頂点 1 にある石を外に取
り出すことを 1 つの操作とする.
たとえば, S1 = 1, S2 = 5, S3 = 17 である.また,1 つの頂点には 2 つ以上の石は置けな
いものとする.
(3) 自然数 k, n (k < n) に対して,次のような条件を満たす自然数の対 (i, j) (i < j) の個数を
k, n で表せ.
条件 :
頂点 i と頂点 j は第 n 段にあり,それぞれの頂点にある石を枝に沿って
1 段ずつ交互に上に動かすと,第 k 段ではじめて出会う.
10.3.4
必要条件・十分条件
02 都立大
1 辺の長さが a の正四面体 ABCD が与えられているとする.
(1) 辺 AC と辺 BD は垂直であることを示せ.
(2) 辺 AC の中点 M と辺 BD の中点 N を結ぶ線分 MN の長さを求めよ.
(3) 直径が b の十分に長い円柱の内部に,正四面体 ABCD がすっぽり入るとき, a, b の満たす
べき不等式を求めよ.ただし,MN の中点 G は円柱の中心軸上にあるものとする.
10.3.5
存在の証明
02 京都府大
→
−
→
座標平面上に原点 O を始点とする n 個のベクトル −
a 1 ,→
a 2 ,· · · ,−
a n をとる (n は自然数).た
→
−
だし,この n 個のベクトルのうち,少なくとも 1 つは 0 でないとする.
−
→
→
これを用いて,座標平面上の原点 O を始点とし,m1 →
a 1 + m2 −
a 2 + · · · + mn −
a n (m1 , m2 , · · · , mn
はすべて任意の整数) の形で表されるベクトルを考える.この形のベクトル全体の集合を M とす
る.このとき次の問いに答えよ.
92
−
→ −
→
(1) M の中より任意に 2 つのベクトル b 1 , b 2 をとる.
→
−
−
→ −
→
−
→
−
→
このときベクトル b 1 + b 2 , b 1 − b 2 , l b 1 (l は任意の整数) も,それぞれ M に属するこ
とを示せ.
(2) 今,M に属するベクトルがすべて原点 O を通るある一直線上に乗っているとする.さらに,
−
→
M に属するベクトルで 0 でないもののうち,その大きさが最小のものが存在するとする.
−
→
そして,大きさが最小のベクトルのうちの 1 つを b とする.このとき,M に属する任意の
−
→
ベクトルは m b (m は整数) と表せることを証明せよ.
10.3.6
定義を問う問題
最近の入試問題では,定義・基本事項を証明させる問題が頻出である.2002 年もいくつか出題
された.気づいたものをここで紹介する.多くは小問の最初の部分である.これらがどのような文
脈で出題されているか知りたければ原題にあたってほしい.
厳格な論述を試してほしい.
• 広島大
n を自然数とする.次の等式が成り立つことを示せ.
(a + b)n = n C0 an + n C1 an−1 b + · · · + n Cn−1 abn−1 + n Cn bn
• 大教大
n
∑
1
(1)
k 2 = n(n + 1)(2n + 1) を証明せよ.
6
k=1
{
}2
n
∑
1
(2)
k3 =
n(n + 1)
を証明せよ.
2
k=1
• 津田塾大
直線 ax + by + c = 0 と点 (x0 , y0 ) の距離を与える公式
|ax0 + by0 + c|
√
a2 + b2
を証明せよ.
• 九州大
原点を中心とする半径 r (r > 0) の円 x2 + y 2 = r2 上の点 (a, b) における接線の方程式は
ax + by = r2 で与えられることを示せ.
• 三重大,宮崎大
logc b
(a > 0, b > 0, c > 0, a ̸= 1, b ̸= 1) を証明せよ.
loga b =
logc a
• 九州大
△ABC の面積は
1
2
√
−→ −→
−→ −→
|AB|2 |AC|2 − (AB · AC)2 に等しいことを示せ.
• 大教大
複素数 z, w に対して,次の不等式が成り立つことを示せ.
|z| − |w| <
= |z| + |w|
= |z + w| <
93
• 南山大
定数 α, β (α < β) に対し
∫
β
(x − α)(x − β) dx = −
α
(β − α)3
6
が成り立つことを示せ.
• お茶の水女子大
区間 a <
=x<
= b を含む範囲で定義された正の値をとる関数 f (x) を考える.y = f (x) のグラ
′
フ,2 直線 x = a, x = t (a <
=t<
= b) で囲まれた部分の面積を S(t) とする. S (t) = f (t) を
示せ.
• 神戸大
関数 f (x) は任意の実数 x に対して定義されているとする.f (x) が x = a において微分可
能であることの定義を述べよ.
• (参)99 東大
(1) 一般角 θ に対して, sin θ, cos θ の定義を述べよ.
(2) (1) で述べた定義にもとづき,一般角 α, β に対して
sin(α + β) = sin α cos β + cos α sin β
cos(α + β) = cos α cos β − cos α cos β
を証明せよ.
• (参) 作問
−
→
−
二つのベクトル →
a = (a1 , a2 ) , b = (b1 , b2 ) がある.また二つのベクトルのなす角を θ と
する.このとき
−
→
−
→
−
→
→
−
−
(1) −
a · b = |→
a || b | cos θ と定めれば,→
a · b = a1 b1 + a2 b2 であることを示せ.
−
→
−
→
−
→
→
→
→
(2) −
a · b = a1 b1 + a2 b2 と定めれば,−
a · b = |−
a || b | cos θ であることを示せ.
10.3.7
背理法
02 東京理科大
(1) 背理法とは何かを 20 字以上 100 字以内で説明せよ.
√
(2) 3 2 が無理数であることを背理法を用いて証明せよ.
02 千葉大
(1) log2 3 は無理数であることを証明せよ.
(2) n が正の整数のとき, log2 n が整数でない有理数となることがあるかどうか調べよ.
02 上智大
半径 1 の球面 A の内側にあり,すべての頂点が球面 A 上にある立方体 B の 1 辺の長さを求め
よ.球面 A の内側で,立方体 B の外側にある最大の球 C の半径を求めよ.球面 A の内側で,立
方体 B と球 C の外側にあり,球 C に接している最大の球 D の半径を求めよ.
94
高校数学の方法関連問題解答
10.4
10.4.1
何をどう置くか
02 名工大
三角形 ABC 複素平面上におき,A(z1 ),B(z2 ), C(z3 ) とする.
α = cos 60◦ + i sin 60◦
とし. D(w1 ), E(w2 ) とおく.点 C ,点 D は直線 AB に関して反対側にあり,また,点 B ,点
E は直線 AC に関して反対側にあるので,
w1 − z2 = α(z1 − Z2 )
w2 − z1 = α(z3 − Z1 )
である.点 M と N に対応する複素数を m, n とすると
w1 + w2
z1 + z2
z3 − z2
=
+α
2
2
2
(
)
(
)
z1 + z2
z3 + z1
一方,K
,L
なので
2
2
m=
n=
2z1 + z2 + z3
4
ここで
n−m =
=
=
{
}
z1 + z2
z3 − z2
2z1 + z2 + z3
−
+α
4
2
2
− z2 + z3
− z2 + z3
−α
4
2
{
(
√ )}
√
− z2 + z3 1
3
3
1
−
+
i
= (z2 − z3 ) ×
i
2
2
2
2
2
n−m
が純虚数になったので,直線 MN と直線 BC が垂直であることが示された.
z2 − z3
10.4.2
条件の同値変形
02 奈良女子大
(1)
a
b
2a−1 <
=2 −2
⇐⇒
a
a−1
= 2a−1
2b <
=2 −2
a−1
b
は成り立つ.
a と b がともに整数で b < a なので b <
=2
= a − 1 が成り立つ.よって 2 <
2b > 0 より 2a − 2b < 2a も成立.
95
(2) (1) から
k
l
k
2k−1 <
=2 −2 <2
m
n
m
2m−1 <
=2 −2 <2
2k − 2l = 2m − 2n なので,
2k−1 < 2m
かつ
2m−1 < 2k
k−1<m
かつ
m−1<k
つまり
ゆえに
k−1<m<k+1
k ,m がともに整数なので k = m である.このとき 2k − 2l = 2m − 2n より l = n である.
[別解] l ̸= n とする.l > n とすれば 2k − 2l = 2m − 2n なので,
2k−n − 2l−n = 2m−n − 1
右辺は奇数で左辺は偶数なので矛盾.
∴
l=n
このとき 2k − 2l = 2m − 2n より k = m .
l < n のときも同様である.
10.4.3
例で考える
02 滋賀医大
(1) 頂点 i が上から k 段目,左から m 番にあるとする.k 段目には 2k−1 個の頂点があるので,
i = 1 + 2 + 22 + · · · + 2k−2 + m = 2k−1 − 1 + m
···⃝
1
が成り立つ.
このとき,頂点 i のすぐ下の段にあって頂点 i と結ばれている頂点は,上から k 段目,左か
ら 2m − 1, 2m の 2 つである.
⃝
1 の関係式の k と m に, k + 1 と 2m − 1, 2m を代入する.
2k+1−1 − 1 + (2m − 1) =
2k+1−1 − 1 + 2m =
2k − 2 + 2m = 2(2k−1 − 1 + m) = 2i
2k+1−1 − 1 + (2m − 1) + 1 = 2i + 1
であるから,それらの頂点の番号は 2i と 2i + 1 である.
96
(2) k 段目の石は k 回の操作で上がりになる.
∴
Sn
n
∑
=
k=1
n
∑
=
k · 2k−1
k · (2k − 2k−1 ) =
k=1
n
∑
{k · 2k − (k − 1) · 2k−1 − 2k−1 }
k=1
= n · 2n −
n
∑
2k−1 = n · 2n − (2n − 1) = (n − 1) · 2n + 1
k=1
(3) k 段目の出会うべき頂点の決め方は 2k−1 通り.
その各々について,第 k 段目のその頂点ではじめて出会うのは,その頂点の下にある k + 1
段目の隣りあう頂点に来ることである.
k + 1 段目の一つの頂点に来る n 段の頂点は 2n−k−1 個あり,頂点 i と頂点 j の決め方がそ
れぞれ 2n−k−1 通りである.
ゆえに求める自然数の対 (i, j) (i < j) の個数は
2k−1 × (2n−k−1 )2 = 22n−k−3 (個)
である.
10.4.4
必要条件・十分条件
02 都立大
(1) 1 辺の長さが a の正四面体 ABCD は 1 辺の長さが
√
2a の立方体のなかに置くことができる.
A
M
A′
C′
C
O
B
N
D
このとき明らかに辺 AC と辺 BD は垂直である.
(2) 辺 AC の中点 M と辺 BD の中点 N を結ぶ線分 MN の長さは,立方体の 1 辺の長さに等し
√
い.ゆえに 2a .
(3) 明らかに線分 MN は辺 AC と辺 BD に垂直である.直線 MN を回転軸にして直径 a の円柱
を作ると正四面体の各頂点がちょうど円柱の表面上にある.
97
したがって,a < b なら,直径が b の十分に長い円柱の内部に,正四面体 ABCD をすっぽ
り入れることができる.ゆえに a < b は十分条件である.
b<
= a なら直径が b の十分に長い円柱の内部に,正四面体 ABCD をすっぽり入れることは
できない.
なぜならもしできたとする.正四面体の中心 O を中心とする球面のうち,この円柱の内部
にくる部分は 2 つある.正四面体の頂点は 4 個あるので,少なくとも一方には 2 つ以上の頂
点が来る.それを A′ , C′ とする.
このとき
̸
A′ OC′ を考えると,これは明らかに
̸
A′ OC′ < ̸ AOC
なので矛盾である.
ゆえに a < b は必要条件でもある.
求める条件は a < b.
10.4.5
存在の証明
02 京都府大
(1)
−
→
b1
−
→
b2
−
→
→
= m1 →
a 1 + m2 −
a 2 + · · · + mn −
an
→
→
−
= l1 −
a 1 + l2 −
a 2 + · · · + ln →
an
とする.このとき
−
→
−
→
b1± b2
−
→
lb1
→
→
→
(m1 ± l1 )−
a 1 + (m2 ± l2 )−
a 2 + · · · + (mn ± ln )−
an
→
→
→
= lm1 −
a 1 + lm2 −
a 2 + · · · + lmn −
an
=
−
→
となり,m1 ± l1 , · · · , mn ± ln ,lm1 , · · · , lmn はすべて整数である.ゆえにベクトル b 1 +
−
→ −
→
−
→
−
→
b 2 , b 1 − b 2 , l b 1 (l は任意の整数) も,それぞれ M に属する.
−
→
→
→
(2) M に属する任意のベクトル −
a をとる.条件から −
a = k b となる実数 k が存在する.また
→
−
p = | b | とおく.p > 0 である.
−
→
−
→
→
−
→
|−
a | = 0,つまり →
a = 0 なら −
a = 0 b と表せ,m = 0 が存在した.
→
−
→
−
−
→
→
|−
a | > 0 とする. b は 0 でないベクトルのなかで絶対値最小なので,| b | <
= | a | である.と
ころが
−
→
−
→
→
|−
a | = |k b | = |k|| b | = |k|p
であるから p <
= |k| である.よってある自然数 m で
= |k|p である.つまり 1 <
m<
= |k| < m + 1
となるものが存在する.
98
k > 0 のとき.
−
→
−
→
−
→
→
c =−
a − m b = (k − m) b
→
とおく.条件から −
c ∈ M で,
→
−c | = (k − m)|−
|→
b | = (k − m)p
−
→
ところが 0 <
= k − m < 1 なので,0 <
= | c | < p である.
→
−
→
ここでもし k − m ̸= 0 なら,|−
c | は絶対値が | b | より小さくかつ 0 でないベクトルになり,
−
→
b が 0 でないベクトルで絶対値最小のものであることと矛盾する.ゆえに k − m = 0.つま
→
−c = −
り→
0 となり,その結果
−
→
−
→
a =mb
と整数 m が存在した.
→
k < 0 のとき.−−
a について同様に
−
→
−
−→
a =mb
−
→
−
と表せるので →
a = (−m) b と表せる.
よって題意が示された.
10.4.6
定義を問う問題
解答は略する.教科書によって確認すること.
10.4.7
背理法
02 東京理科大
(1) 結論の否定を仮定し,何らかの矛盾が生じることを示す.それによって結論の否定が否定さ
れ,結論が成立することを示す証明方法.
√
(2) 3 2 が有理数であると仮定する.
√
q
[証明 1] 3 2 = (p と q は互いに素な整数) とおく.
p
2p3 = q 3
となるので, q は偶数. q = 2q ′ とおく.このとき
2p3 = 8q ′
3
より p も偶数となり,互いに素という仮定と矛盾した.
√
ゆえに 3 2 は無理数.
99
√
q
[証明 2] 3 2 = (p と q は整数) とおく.
p
2p3 = q 3
となる.左辺の因数分解における因数 2 の個数は 3 で割ると 1 余る.それに対して右辺
の因数分解における因数 2 の個数は 3 の倍数.素因数分解の一意性と矛盾した.
√
ゆえに 3 2 は無理数.
√
q
[証明 3] 3 2 = (p と q は 0 でない整数) とおく.
p
2p3 = q 3
となるので, q は偶数. q = 2q ′ とおく.このとき
2p3 = 8q ′
3
より p も偶数. p = 2p′ とおく.これを代入して約すると
2p′ = q ′
3
3
再び p′ , q ′ が 2 で約せ,これは何回でも繰り返せる.つまり p, q とも 2 で無限回約せ
る.これは p と q は 0 でない整数という仮定と矛盾した.
√
ゆえに 3 2 は無理数.
02 千葉大
(1) log2 3 が有理数と仮定し
log2 3 =
p
(p, q は互いに素,q > 0 )
q
p
とおく.このとき,2 q = 3 なので
2p = 3q
素因数分解の一意性によりこの式は矛盾である.ゆえに log2 3 は無理数である.
(2) ある正の整数 n に対して, log2 n が整数でない有理数とする.
log2 n =
p
(p, q は互いに素で p ̸= 0, q > 1 )
q
とおく.このとき同様に
2 p = nq
素因数分解の一意性により n = 2r となる非負整数 r が存在する.このとき指数を比較して
p = rq
これは p, q が互いに素であることに矛盾した.ゆえに log2 n が整数でない有理数と なるこ
とはない.
100
02 上智大
r2
r1
l
√
2l
求める立方体 B の 1 辺の長さを l とする.立方体の対角線での断面を考える.
√
l2 + ( 2l)2 = 2
2
より, l = √
3
次に球 C の半径を r1 とする.明らかに
√
(
)
1
l
3− 3
r1 =
1−
=
2
2
6
さらに球 D の半径を r2 とする.
(1 − r2 )2 − (1 − 2r1 + r2 )2 = (r1 + r2 )2 − (r1 − r2 )2
これから
r2 = r1 (1 − r1 ) =
101
1
6